Sei sulla pagina 1di 72

REPUBLIC V.

DEL MONTE MOTORS


FACTS: case involves a petition seeking the reversal of the trial court decision which found Comm.
Malinis guilty of indirect contempt when he refused to obey a certain resolution. On Jan. 15-02, the
RTC rendered a decision finding Vilfran Liner Inc. and the Vilegas’s to be solidary liable to Del Monte
Motors. The trial court ordered the execution of the decision against the counterbond posted by
Vilfran Liner Inc. and issued by Capital Insurance and Surety Co Inc. On April 18, 2002, CISCO opposed
the Motion for Execution filed by respondent, claiming that the latter had no record or document
regarding the alleged issuance of the counterbond; thus, the bond was not valid and enforceable.
On June 13, 2002, the RTC granted the Motion for Execution and issued the corresponding Writ.
Armed with this Writ, Sheriff Manuel S. Paguyo proceeded to levy on the properties of CISCO. He also
issued a Notice of Garnishment on several depository banks of the insurance company. Moreover, he
served a similar notice on the Insurance Commission, so as to enforce the Writ on the security deposit
filed by CISCO with the Commission in accordance with Section 203 of the Insurance Code.
On December 18, 2002, after a hearing on all the pending Motions, the RTC ruled that the Notice of
Garnishment served by Sheriff Paguyo on the insurance commission was valid. On January 8, 2003,
respondent moved to cite Insurance Commissioner Eduardo T. Malinis in contempt of court for his
refusal to obey the December 18, 2002 Resolution of the trial court. - The resolution dated December
18, 2002 directed him to allow the withdrawal of the security deposit of Capital Insurance and Surety
Co. (CISCO) in the amount of P11,835,375.50 RTC Decision: Malinis found to be in contempt of court.
ISSUE: W/N THE SECURITY DEPOSIT HELD BY THE INSURANCE COMMISSIONER PURSUANT TO SECTION
203 OF THE INSURANCE CODE MAY BE LEVIED OR GARNISHED IN FAVOR OF ONLY ONE INSURED? -
RULING: NO. Contrary to respondent’s contentions that Section 203 does not provide for an absolute
prohibition on the levy and garnishment of the security deposit, the Court opined that the law
expressly and clearly states that the security deposit shall be (1) answerable for all the obligations of
the depositing insurer under its insurance contracts; (2) at all times free from any liens or
encumbrance; and (3) exempt from levy by any claimant.
To be sure, CISCO, though presently under conservatorship, has valid outstanding policies. Its policy
holders have a right under the law to be equally protected by its security deposit. To allow the
garnishment of that deposit would impair the fund by decreasing it to less than the percentage of
paid-up capital that the law requires to be maintained. Further, this move would create, in favor of
respondent, a preference of credit over the other policy holders and beneficiaries.
Our Insurance Code is patterned after that of California. Thus, the ruling of the state's Supreme Court
on a similar concept as that of the security deposit is instructive. Engwicht v. Pacific States Life
Assurance Co. held that the money required to be deposited by a mutual assessment insurance
company with the state treasurer was "a trust fund to be ratably distributed amongst all the claimants
entitled to share in it. Such a distribution cannot be had except in an action in the nature of a
creditors' bill, upon the hearing of which, and with all the parties interested in the fund before it, the
1
court may make equitable distribution of the fund, and appoint a receiver to carry that distribution
into effect." - A single claimant may not lay stake on the securities to the exclusion of all others. The
other parties may have their own claims against the insurance company under other insurance
contracts it has entered into - The right to lay claim on the fund is dependent on the solvency of the
insurer and is subject to all other obligations of the company arising from its insurance contracts.
Thus, respondent's interest is merely inchoate. Being a mere expectancy, it has no attribute of
property. At this time, it is nonexistent and may never exist.

GULF RESORT,INC VS PHILIPPINE CHARTER INSURANCE CORPORATION


G.R. No. 156167 May 16, 2005
FACTS :
Gulf Resorts is the owner of the Plaza Resort situated at Agoo, La Union
and had its properties in said resort insured originally with the American
Home Assurance Company (AHAC). In the first 4 policies issued, the
risks of loss from earthquake shock was extended only to petitioner’s
two swimming pools. Gulf Resorts agreed to insure with Phil Charter the
properties covered by the AHAC policy provided that the policy wording
and rates in said policy be copied in the policy to be issued by Phil
Charter. Phil Charter issued Policy No. 31944 to Gulf Resorts covering
the period of March 14, 1990 to March 14, 1991 for P10,700,600.00 for a
total premium of P45,159.92. the break-down of premiums shows that
Gulf Resorts paid only P393.00 as premium against earthquake shock
(ES). In Policy No. 31944 issued by defendant, the shock endorsement
provided that “In consideration of the payment by the insured to the
company of the sum included additional premium the Company agrees,
notwithstanding what is stated in the printed conditions of this policy due
to the contrary, that this insurance covers loss or damage to shock to
any of the property insured by this Policy occasioned by or through or in
consequence of earthquake. In Exhibit "7-C" the word "included" above
the underlined portion was deleted. On July 16, 1990 an earthquake
struck Central Luzon and Northern Luzon and plaintiff’s properties
covered by Policy No. 31944 issued by defendant, including the
two swimming pools in its Agoo Playa Resort were damaged.
Petitioner advised respondent that it would be making a claim under its
Insurance Policy 31944 for damages on its properties. Respondent
denied petitioner’s claim on the ground that its insurance policy only
afforded earthquake shock coverage to the twoswimming pools of the
resort. The trial court ruled in favor of respondent. In its ruling,
the schedule clearly shows that petitioner paid only a premium of
P393.00 against the peril of earthquake shock, the same premium it had
paid against earthquake shock only on the two swimming pools in all the
policies issued by AHAC.
2
ISSUE: Whether or not the policy covers only the two swimming
pools owned by Gulf Resorts and does not extend to all properties
damaged therein
HELD: Yes, All the provisions and riders taken and interpreted together,
indubitably show the intention of the parties to extend earthquake shock
coverage to the two swimming pools only. Aninsurance premium is the
consideration paid an insurer for undertaking to indemnify the insured
against a specified peril. In fire, casualty and marine insurance, the
premium becomes a debt as soon as the risk attaches. In the subject
policy, no premium payments were made with regard to earthquake
shock coverage except on the two swimming pools. There is no mention
of any premium payable for the other resort properties with regard to
earthquake shock. This is consistent with the history of
petitioner’sinsurance policies with AHAC.

PHILAMCARE HEALTH SYSTEMS, INC. vs. COURT OF APPEALS G.R. No. 125678, 18 March 2002
FACTS: Ernani Trinos obtained a health care coverage with petitioner Philamcare. Under the
agreement, Trinos was entitled to avail of hospitalization benefits, whether ordinary or emergency,
listed therein. He was entitled to avail of “out-patient benefits” such as annual physical examinations,
preventive health care and other out-patient services. During the period of coverage, Trinos suffered a
heart attack and was hospitalized for one month. During this time, his wife, Julita Trinos, tried to claim
the benefits under the health care agreement but petitioner company denied her claim on the ground
that the Health Care Agreement was void because there was concealment regarding Ernani’s medical
history. Doctors allegedly discovered at the time of Ernani’s confinement that he was hypertensive,
diabetic and asthmatic, contrary to his answer in the application form. Thus Julita paid the
hospitalization expenses herself. When Ernani died, Julita instituted with the RTC of Manila an action
for damages against petitioner and its president. She asked for reimbursement of her expenses plus
moral damages and attorney’s fees.
ISSUE: Whether or not the petitioner is liable
HELD: YES. The health care agreement was in the nature of non-life insurance, which is primarily a
contract of indemnity. Once the
member incurs hospital, medical or any other expense arising from sickness, injury or other stipulated
contingent, the health care provider must pay for the same to the extent agreed upon under the
contract. Petitioner alleges that respondent was not the legal wife of the deceased member
considering that at the time of their marriage, the deceased was previously married to another
woman who was still alive. The health care agreement is in the nature of a contract of indemnity.
Hence, payment should be made to the party who incurred the expenses. It is not controverted that
respondent paid all the hospital and medical expenses. She is therefore entitled to reimbursement.
The records adequately prove the expenses incurred by respondent for the deceased’s hospitalization,
3
medication and the professional fees of the attending physicians. WHEREFORE, in view of the
foregoing, the petition is DENIED. The assailed decision of the Court of Appeals dated December 14,
1995 is AFFIRMED.

Heirs of Ildefonso Coscolluela, Sr., INC., petitioner


vs Rico General Insurance Corporation, Court of Appeals, etc., respondents

Facts:

Heirs, is a domestic corporation and registered owner of an Isuzu pick-up truck which was insured with
Rico General Insurance for a consideration P100,000 excluding 3rd party liability. The premiums and
other expenses for insurance paid covered the period from October 1, 1986 to 1987.

On August 28, 1987, the insured vehicle was severely damaged when fired upon by unidentified
armed persons in Negros Occidental. In the same incident, four persons died.

Heirs then filed its claim for the repair of the vehicle but Rico refused t grant it. So heirs filed it with
the RTC Bacolod City to recover claim, plus interest and attorney's fees. Rico filed a motion to dismiss
alleging that it lacks cause of action because firing by armed men is a risk excepted in the insurance
policy.

Heirs alleged that the firing was an indirect consequence of rebellion and opposed the motion to
dismiss saying that the provision does not apply in the absence of an official proclamation of the
conditions. RTC dismissed the complaint of Heirs fo lack of cause of action for the damage arose from
a civil commotion or was a direct result thereof.

Issue: Whether CA erred in affirming the dismissal by the trial court of the complaint for damages in
the ground of lack of cause of action

Held:
(1) There is cause of action. The elements were met.

The facts as alleged clearly define the existence of a right of the petitioner to a just claim against the
insurer for the payment of the indemnity for a loss due to an event against which the petitioner's
vehicle was insured. The insurance contract mentioned therein manifests a right to pursue a claim and
a duty on the part of the insurer or private respondent to compensate the insured in case of a risk
insured against. The refusal of the insurer to satisfy the claim and the consequent loss to the
petitioner in incurring the cost of acquiring legal assistance on the matter constitutes a violation or an
injury brought to the petitioner.

4
Piltel vs. Tecson G. R. No. 156966 May 7, 2004

Facts:

Tecson applied for six cellular phone subscriptions from Piltel, and each was covered by 6 agreements.
Failure to comply with his obligation, Tecson instituted an action for collection of sum of money plus
damages against Piltel in the RTC of Iligan City. Piltel moved to dismiss the complaint on the ground of
improper venue. It alleges that it was agreed upon by them that the “Venue of all suits arising from
this Agreement or any other suit directly or indirectly arising from the relationship between PILTEL
and subscriber shall be in the proper courts of Makati, Metro Manila. Subscriber hereby expressly
waives any other venues.” RTC denied the motion to dismiss. CA affirmed, its ruling appearing to be
anchored on the thesis that the agreement, being a mere contract of adhesion, does not bind Tecson
on the venue stipulation.

Issue: WON the motion to dismiss should be granted on the ground of improper venue.

Held:

Yes.

A contract duly executed is the law between the parties, and they are obliged to comply fully and not
selectively with its terms. A contract of adhesion is no exception.

Thus, the provision contained in the Agreement, a standard contract made out by petitioner PILTEL to
its subscribers, apparently accepted and signed by Tecson, states that the venue of all suits arising
from the agreement, or any other suit directly or indirectly arising from the relationship between
PILTEL and subscriber, “shall be in the proper courts of Makati, Metro Manila.” The added stipulation
that the subscriber “expressly waives any other venue"should indicate, clearly enough, the intent of
the parties to consider the venue stipulation as being preclusive in character.

Fortune Insurance and Surety Co., Inc., vs. CA [G.R. No. 115278, May 23, 1995]

Facts: On June 29, 1987, Producer’s Bank of the Philippines’armored vehicle was robbed, in transit, of
seven hundred twenty-five thousand pesos (Php 725,000.00) that it was transferring from its branch in
Pasay to its main branch in Makati. To mitigate their loss, they claim the amount from their insurer,
namely Fortune Insurance and Surety Co..
Fortune Insurance, however, assails that the general exemptionclause in the Casualty Insurance
coverage had a general exemptionclause, to wit:

5
GENERAL EXCEPTIONS
The company shall not be liable under this policy in respect of
xxx xxx xxx
(b) any loss caused by any dishonest, fraudulent or criminal act of the insured or any officer,
employee, partner, director, trustee or authorized representative of the Insured whether acting alone
or in conjunction with others. . . .
And, since the driver (Magalong) and security guard (Atiga) of the armored vehicle were charged with
three others as liable for the robbery, Fortune denies Producer’s Bank of its insurance claim.
The trial court and the court appeals ruled in favor of recovery, hence, the case at bar.

Issue: Whether recovery is precluded under the general exemption clause.


Held: Yes, recovery is precluded under the general exemption clause. Howsoever viewed, Producers
entrusted the three with the specific duty to safely transfer the money to its head office, with
Alampay to be responsible for its custody in transit; Magalong to drive the armored vehicle which
would carry the money; and Atiga to provide the needed security for the money, the vehicle, and his
two other companions. In short, for these particular tasks, the three acted as agents of Producers. A
"representative" is defined as one who represents or stands in the place of another; one who
represents others or another
in a special capacity, as an agent, and is interchangeable with "agent." 23
In view of the foregoing, Fortune is exempt from liability under the general exceptions clause of the
insurance policy.

Calanoc v. CAG.R. No. L-8151 December 16, 1955J.


Doctrine: In case of ambiguity in an insurance contract covering accidental death, the Supreme
Courtheld that such terms shall be construed strictly against the insurer and liberally in favor of the
insured Inorder to effect the purpose of indemnity.
Facts: Melencio Basilio, a watchman of the Manila Auto Supply, secured a life insurance policy fromthe
Philippine American Insurance Company in the amount of P2,000 to which was attached
asupplemental contract covering death by accident. He later died from a gunshot wound on the
occasionof a robbery committed; subsequently, his widow was paid P2,000 representing the face
value of thepolicy. The widow demanded the payment of the additional sum of P2,000 representing
the value of thesupplemental policy which the company refused because the deceased died by

6
murder during therobbery and while making an arrest as an officer of the law which were expressly
excluded in the contract. The company’s contention which was upheld by the Court of Appeals
provides that the circumstances surrounding Basilio’s death was caused by one of the risks excluded
by the supplementary contract which exempts the company from liability.
Issue: Is the Philippine American Life Insurance Co. liable to the petitioner for the amount covered by
thesupplemental contract?
Held: Yes. The circumstances of Basilio’s death cannot be taken as purely intentional on the part of
Basilio to expose himself to the danger. There is no proof that his death was the result of
intentionalkilling because there is the possibility that the malefactor had fired the shot merely to scare
away the people around. In this case, the company’s defense points out that Basilio’sis included
among the risksexcluded in the supplementary contract; however, the terms and phraseology of the
exception clauseshould be clearly expressed within the understanding of the insured. Art. 1377 of the
New Civil Codeprovides that in case ambiguity, uncertainty or obscurity in the interpretation of the
terms of thecontract, it shall be construed against the party who caused such obscurity.
Applying this to thesituation, the ambiguous or obscure terms in the insurance policy are to be
construed strictly against theinsurer and liberally in favor of the insured party. The reason is to ensure
the protection of the insuredsince these insurance contracts are usually arranged and employed by
experts and legal advisers actingexclusively in the interest of the insurance company. As long as
insurance companies insist upon the useof ambiguous, intricate and technical provisions, which
conceal their own intentions, the courts must, infairness to those who purchase insurance, construe
every ambiguity in favor of the insured.

DIOSDADO C. TY vs. FIRST NATIONAL SURETY & ASSURANCE CO. INC. G.R. NO. L-16138, April 29, 961 1
SCRA 1324
Facts: Petitioner obtained personal accident policies which stipulated, among others, that for partial
disability resulting to the loss of either hand, the insurer shall be liable for P650.00. It was further
stated in the policies that, “That loss of a hand shall mean the loss by amputation through the bones
of the wrist.” A fire broke out which totally destroyed Broadway Cotton Factory, Ty’s employer. Fighting
his way out of the factory, Ty was injured on the left hand by a heavy object. As a result, Ty suffered a
temporary total disability of his left hand which prevented hi from performing his work or labor
necessary in the pursuance of his occupation.
Issue: Whether or not the insurer is liable
Held: The insurer was not liable. We can not go beyond the clear and express conditions of the
insurance policies, all of which defined partial disability as loss of either hand by amputation through
the bones of the wrist. There was no such amputation. All that was found was that the physical
injuries caused temporary total disability of Ty’s left hand. We might add that the agreement
contained in the insurance policies are clear, express and specific that only amputation of the left hand
7
should be considered as a loss thereof, an interpretation that would include the mere fracture or
other temporary disability not covered by the policies would certainly be unwarranted. WHEREFORE,
the decision appealed from is hereby affirmed, with costs against the plaintiff-appellant.

SIMON DE LA CRUZ vs. THE CAPITAL INSURANCE AND SURETY CO., INC. G.R. No. L-21574, June 30,
1966 17 SCRA 599
FACTS: Eduardo de la Cruz, the son of herein petitioner, was the holder of an accident insurance policy.
In connection with the celebration of the New Year, the insured, a non-professional boxer, participated
in a boxing contest. In the course of his bout with another person, likewise a non-professional, of the
same height, weight, and size, Eduardo slipped and was hit by his opponent on the left part of the
back of the head, causing Eduardo to fall, with his head hitting the rope of the ring. The insured died
with the cause of death reported as “hemorrhage intercranial, left”. The insurer refused to pay the
proceeds of the policy on the ground that the death of the insured, caused by his participation in a
boxing contest, was not accidental and, therefore, not covered by insurance.
ISSUE: Whether or not the death of the insured is covered by the policy
HELD: The terms “accident” and “accidental” as used in the insurance contract, have not acquired any
technical meaning, and are construed by the courts in their ordinary and common acceptation. Thus,
the terms have been taken to mean that which happen by chance or fortuitously, without intention
and design, and which is unexpected, unusual, and unforeseen. An accident is an event that proceeds
from an unknown cause and, therefore, not expected. Without the unintentional slipping of the
deceased, perhaps he would not have received the blow in the head and would not have died. Boxing
is attended with some risks of external injuries, but any injury received in the course of the game
could be accidental. In boxing, as in other equally physically rigorous sports, such as basketball or
baseball, death is not ordinarily anticipated to result. If, therefore, it ever does, the injury or death can
only be accidental or produced by some unforeseen happening or event as what occurred in this case.
The insurer was liable.

VIRGINIA A. PEREZ, petitioner, vs. COURT OF APPEALS and BF LIFEMAN INSURANCE CORPORATION,
respondents. G.R. No. 112329 January 28, 2000 YNARES-SANTIAGO, J.
Facts: Primitivo Perez has been isured with the BF Lifeman Insurance Corporation since 1980 for
P20,000. Sometime in 1987, Rodolfo Lalog, an agent of BF, convinced him to apply for additional
insurance coverage of P50, 000. Perez accomplished the application form and passed the required
medical exam. He also paid P2,075 to Lalog for premium. On Nov. 25, 1987, perez died while riding a
banca which capsized during a storm. During this time his application papers for the additional
insurance coverage was still in the office of BF. Without knowing that Perez died, BF approved Perez’s
application and issued the corresponding policy for P50,000. Virginia Perez, his wife, claimed the
benefits of the insurance policy for her deceased husband but she was only able to obtain P40,000
8
under the first insurance policy. BF refused to pay the proceeds amounting to P150,000 under the
additional policy coverage of P50,000 because they maintain that such policy had not been perfected.
On Sept. 21, 1990, BF filed a complaint against Mrs. Perez seeking recission and declaration of nullity
of the insurance contract in question. Mrs. Perez file a counterclaim for the collection of P150,000 plus
damages.
Issue: Whether or not there was a consummated contract of insurance between Perez and BF.
Held: No. An essential requisite of a valid contract is consent. Consent must be manifested by the
meeting of the offer and acceptance upon the thing and the cause which are to constitute the
contract. The offer must be certain and the acceptance absolute. When Perez filed the application , it
was subject to the acceptance of BF. The perfection was also further conditioned upon 1) issuance of
the policy; 2) payment of the premium and; 3) the delivery to and acceptance by the applicant in good
health. The delivery and acceptance by the applicant was a suspensive condition which was not
fulfilled inasmuch as the applicant was already dead at the time the policy was issued. The non-
fulfillment of the condition resulted in the non-perfection of the contract. An application for insurance
is merely an offer which requires the overt act of the insurer for it to ripen to a contract. Delay in
acting on the application does not constitute acceptance even though the insured has forwarded his
first premium with his application. Delay, in this case, does not constitute gross negligence because
the application was granted within the normal processing time. WHEREFORE, the decision rendered
by the Court of Appeals in CA-G.R. CV No. 35529 is AFFIRMED insofar as it declared Insurance Policy
No. 056300 for P50,000.00 issued by BF Lifeman Insurance Corporation of no force and effect and
hence null and void. No costs.

De lim LIM V. SUN LIFE gr 15774


Facts: > On July 6, 1917, Luis Lim Y Garcia of Zamboanga applied for a policy of life insurance with
Sunlife in the amount of 5T. > He designated his wife Pilar Lim as the beneficiary. The first premium of
P433 was paid by Lim and company issued a ―provisional policy‖ > Such policy contained the
following provisions “xx the abovementioned life is to be assured in accordance with the terms and
conditions contained or inserted by the Company in the policy which may be granted by it in this
particular case for 4 months only from the date of the application, PROVIDED that the company shall
confirm this agreement by issuing a policy on said application xxx. Should the company NOT issue such
a policy, then this agreement shall be null and void ab initio and the Company shall be held not to
have been on the risk at all, but in such case, the amount herein shall be returned. > Lim died on Aug.
23, 1917 after the issuance of the provisional policy but before the approval of the application by the
home office of the insurance company. > The instant action is brought by the beneficiary to recover
from Sun Life the sum of 5T.

Issue: Whether or not the beneficiary can collect the 5T.


9
Held: NO. The contract of insurance was not consummated by the parties. The above quoted
agreement clearly stated that the agreement should NOT go into effect until the home office of the
Company shall confirm it by issuing a policy. It was nothing but an acknowledgment by the Company
that it has received a sum of money agreed upon as the first year’s premium upon a policy to be
issued upon the application if it is accepted by the Company.

When an agreement is made between the applicant and the agent whether by signing an application
containing such condition or otherwise, that no liability shall attach until the principal approves the
risk and a receipt is given by the agent, such acceptance is merely conditional and is subordinated to
the company’s act in approving or rejecting; so in life insurance a ―binding slip or receipt‖ does not
insure itself.

Development Bank of the Philippines v CA 231 SCRA 370 March 21, 1994 GR. 109937
Facts: Juan B. Dans, together with his family applied for a loan of P500,000 with DBP. As principal
mortgagor, Dans, then 76 years of age was advised by DBP to obtain a mortgage redemption
insurance (MRI) with DBP MRI pool. A loan in the reduced amount was approved and released by DBP.
From the proceeds of the loan, DBP deducted the payment for the MRI premium. The MRI premium
of Dans, less the DBP service fee of 10%, was credited by DBP to the savings account of DBP MRI-Pool.
Accordingly, the DBP MRI Pool was advised of the credit. Dans died of cardiac arrest. DBP MRI Pool
notified DBP that Dans was not eligible for MRI coverage, being over the acceptance age limit of 60
years at the time of application. DBP apprised Candida Dans of the disapproval of her late husband’s
MRI application. DBP offered to refund the premium which the deceased had paid, but Candida Dans
refused to accept the same demanding payment of the face value of the MRI or an amount
equivalent of the loan. She, likewise, refused to accept an ex gratia settlement which DBP later
offered. Hence the case at bar.
Issue: Whether or not the DBP MRI Pool should be held liable on the ground that the contract was
already perfected?
Held: No, it is not liable. The power to approve MRI application is lodged with the DBP MRI Pool. The
pool, however, did not approve the application. There is also no showing that it accepted the sum
which DBP credited to its account with full knowledge that it was payment for the premium. There
was as a result no perfected contract of insurance’ hence the DBP MRI Pool cannot be held liable on a
contract that does not exist In dealing with Dans, DBP was wearing 2 legal hats: the first as a lender
and the second as an insurance agent. As an insurance agent, DBP made Dans go through the motion
of applying for said insurance, thereby leading him and his family to believe that they had already
fulfilled all the requirements for the MRI and that the issuance of their policy was forthcoming. DBP
10
had full knowledge that the application was never going to be approved. The DBP is not authorized to
accept applications for MRI when its clients are more than 60 years of age. Knowing all the while that
Dans was ineligible, DBP exceeded the scope of its authority when it accepted the application for MRI
by collecting the insurance premium and deducting its agent’s commission and service fee. Since the
third person dealing with an agent is unaware of the limits of the authority conferred by the principal
on the agent and he has been deceived by the nondisclosure thereof by the agent, then the latter is
liable for damages to him.

Enriquez vs. Sun Life Assurance Co. [GR No. 15895; November 29, 1920]
Facts: Plaintiff is estate administrator for late Joaquin Herrer. Herrer has pending application with
defendant Sun Life Assurance Co (sun Life) evidenced by a provisional receipt. The provisional receipt
reads payment of Php6, 000 for life annuity received 26 September 1917. The application was
received by Sun Life head office a month after. 04 December 1917, the policy was issued in Montreal.
A petition for withdrawal of application was filed by Herrer’s lawyer 18 December 1917. Herrer died
20 December. A letter from Sun Life was received 21 December stating policy was issued and reminds
the party of a notification of acceptance of the application dated 26 November. Plaintiff testified that
he had found no letter of notification from the Sun Life. Lower Court decides in favor of respondent.
Appeal was taken.
Issue: Whether or not the there has been a valid offer and acceptance??
Held: None. The Civil Code provides that the acceptance made by letter binds the person making the
offer only from the date it has came to its knowledge. The contract of life annuity was not perfected.
There was no satisfactory evidence that the application acceptance came to the knowledge of Herrer.
Article 16 of the civil code provides that any deficiency in the special law shall be supplied by the
Code. The Insurance Code does not provide for law on the principle of acceptance, thus the Civil Code
shall govern. Article 1262 provides that consent is shown by concurrence of offer and acceptance
with the thing and the consideration to the contract. The acceptance by letter shall not bind the
person making the offer except from the time It came to his knowledge. American Courts held that
acceptance of offer not actually communicated does not complete the contract but the mailing of the
acceptance. Locus Poenitrntiae is ended when acceptance has passed beyond party’s control.
Furthermore, the provisional receipt provides for conditions before a contract is deemed final. 1.
Medical examination. 2. Approval by head office of the application. 3. the company communicates
approval to the applicant. In the case, there was no letter of notification. No evidence of knowledge.
Judgment reversed. Php6000 with interest is to be returned.

K. S. YOUNG vs. THE MIDLAND TEXTILE INSURANCE COMPANY

11
[G.R. No. 9370; March 31, 1915]

Warranties: violation of material warranty, effect

Facts:

The purpose of the present action is to recover the sum of P3,000 upon an insurance policy. The lower
court rendered a judgment in favor of the plaintiff and against the defendant for the sum
of P2,708.78, and costs. From that judgment the defendant appealed to this court. The undisputed
facts upon which said action is based are as follows: The plaintiff occupied a building at ‘321 Calle
Claveria, as a residence and bodega (storehouse).On the 29th of May, 1912, the defendant, in
consideration of the payment of a premium of P60, entered into a contract of insurance with the
plaintiff promising to pay to the plaintiff the sum of P3,000, in case said residence and bodega and
contents should be destroyed by fire. One of the conditions of said contract was that no hazardous
goods be stored or kept in the building. On the 4th or 5th of February, 1913, the plaintiff placed in said
residence and bodega three boxes which belonged to him and which were filled with fireworks for the
celebration of the Chinese new year. On the 18th day of March, 1913, said residence and bodega and
the contents thereof were partially destroyed. Fireworks were found in a part of the building not
destroyed by the fire; that they in no way contributed to the fire, or to the loss occasioned thereby.

Issue:

Whether or not the placing of said fireworks in the building insured, under the conditions above
enumerated, they being “hazardous goods,” is a violation of the terms of the contract of insurance.

Ruling:

Yes. The word “stored” has been defined to be a deposit in a store or warehouse for preservation or
safe keeping; to put away for future use, especially for future consumption; to place in a warehouse or
other place of deposit for safe keeping. Said definition does not include a deposit in a store, in small
quantities, for daily use. “Daily use” precludes the idea of deposit for preservation or safe keeping, as
well as a deposit for future consumption or safe keeping. A violation of the terms of a contract of
insurance, by either party, will constitute the basis for a termination of the contractual relations, at
the election of the other. The right to terminate the contractual relations exists even though the
violation was not the direct cause of the loss. In the present case, the deposit of the “hazardous
goods,” in the building insured, was a violation of the terms of the contract. Although the hazardous
goods did not contribute to the loss, the insurer, at his election, was relieved from liability Said deposit
created a new risk, not included in the terms of the contract. The insurer had neither been paid, nor
had he entered into a contract, to cover the increased risk.

ANG GIOK CHIP vs. SPRINGFIELD

[G.R. No. L-33637; December 31, 1931]


12
Facts:

Ang insured his warehouse for the total value of Php 60,000. One of these, amounting to 10,000, was
with Springfield Insurance Company. His warehouse burned down, then he attempted to recover
8,000 from Springfield for the indemnity. The insurance company interposed its defense on a rider in
the policy in the form of Warranty F, fixing the amount of hazardous good that can be stored in a
building to be covered by the insurance. They claimed that Ang violated the 3 percent limit by placing
hazardous goods to as high as 39 percent of all the goods stored in the building. His suit to recover
was granted by the trial court. Hence, this appeal.

Issue:

Whether a warranty referred to in the policy as forming part of the contract of insurance and in the
form of a rider to the insurance policy, is null and void because not complying with the Philippine
Insurance Act.

Ruling:

No. The warranty is valid. Petition dismissed.

Ratio:

The Insurance Act, Section 65, taken from California law, states:

“Every express warranty, made at or before the execution of a policy, must be contained in the policy
itself, or in another instrument signed by the insured and referred to in the policy, as making a part of
it.”

Warranty F, indemnifying for a value of Php 20,000 and pasted on the left margin of the policy stated:

It is hereby declared and agreed that during the currency of this policy no hazardous goods be stored
in the Building to which this insurance applies or in any building communicating therewith, provided,
always, however, that the Insured be permitted to stored a small quantity of the hazardous goods
specified below, but not exceeding in all 3 per cent of the total value of the whole of the goods or
merchandise contained in said warehouse, viz; . . . .

Also, the court stated a book that said, “any express warranty or condition is always a part of the
policy, but, like any other part of an express contract, may be written in the margin, or contained in
proposals or documents expressly referred to in the policy, and so made a part of it.”

“It is well settled that a rider attached to a policy is a part of the contract, to the same extent and with
like effect as it actually embodied therein. In the second place, it is equally well settled that an express
warranty must appear upon the face of the policy, or be clearly incorporated therein and made a part
thereof by explicit reference, or by words clearly evidencing such intention.”

13
The court concluded that Warranty F is contained in the policy itself, because by the contract of
insurance agreed to by the parties it was made to be a part. It wasn’t aseparate instrument agreed to
by the parties.

The receipt of the policy by the insured without objection binds him. It was his duty to read the policy
and know its terms. He also never chose to accept a different policy by considering the earlier one as a
mistake. Hence, the rider is valid.

American Home Assurance v. Tantuco

G.R. No. 138941, 8 Oct. 2001

 INSURANCE LAW: Liberality is the rule of construction in insurance contracts.

FACTS:

Tantuco Enterprises, Inc. is a coconut oil milling and refining company. It owned two mills (the first oil
mill and a new one), both located at its factory compound at Iyam, Lucena City. The two oil mills are
separately covered by fire insurance policies issued by American Home Assurance Co.

On Sept. 30, 1991, a fire broke out and gutted and consumed the new oil mill. American Home
rejected the claim for the insurance proceeds on the ground that no policy was issued by it covering
the burned oil mill. It stated that the new oil mill was under Building No. 15 while the insurance
coverage extended only to the oil mill under Building No. 5.

ISSUE:
 Whether or not the new oil mill is covered by the fire insurance policy

HELD:

In construing the words used descriptive of a building insured, the greatest liberality is shown by the
courts in giving effect to the insurance. In view of the custom of insurance agents to examine buildings
before writing policies upon them, and since a mistake as to the identity and character of the building
is extremely unlikely, the courts are inclined to consider the policy of insurance covers any building
which the parties manifestly intended to insure, however inaccurate the description may be.

Notwithstanding, therefore, the misdescription in the policy, it is beyond dispute, to our mind, that
what the parties manifestly intended to insure was the new oil mill.
14
If the parties really intended to protect the first oil mill, then there is no need to specify it as new.
Indeed, it would be absurd to assume that the respondent would protect its first oil mill for different
amounts and leave uncovered its second one.

QUA CHEE GAN vs. LAW UNION AND ROCK INSURANCE CO., LTD. G.R. No. L-4611, 17 December 1955
FACTS: Plaintiff-appellee owned four bodegas used for the storage of copra and hemp. These
buildings, together with their contents, were insured with the defendant company since 1937 and the
loss made payable to the Philippine National Bank as mortgage of the hemp and crops, to the extent
of its interest.
Sometime after, three of the bodegas, together with the merchandise inside, were completely
destroyed by fire of an undetermined origin. Consequently, Qua Chee Gan notified the insurance
company of his loss. The latter conducted an extensive investigation. The damage was determined to
be equivalent to P398,562.81 which was later reduced to the full amount of the insurance,
Php370,000.00. However, the insurance company refused payment, claiming violation of warranties
and conditions, filing of fraudulent claims, and that the fire had been deliberately caused by the
insured or by other persons in connivance with him. If moreover argued that since the bodegas
insured had an external wall perimeter of 500 meters or 1,640 feet, the appellee should have 11 fire
hydrants in the compound, and that he actually had only 2 with a further pair nearby, belonging to the
municipality of Tabaco.
ISSUE: Whether or not the insurer company is liable
HELD: YES. The SC is in agreement with the trial court that the appellant is barred by waiver (or rather
estoppel) to claim violation of the so-called fire hydrants warranty, for the reason that knowing fully all
that the number of hydrants demanded therein never existed from the very beginning, the appellant
nevertheless issued the policies in question subject to such warranty, and received the corresponding
premiums. It would be perilously close to conniving at fraud upon the insured to allow appellant to
claims now as void ab initio the policies that it had issued to the plaintiff without warining of their
fatal defect, of which it was informed, and after it had misled the defendant into believing that the
policies were effective. The contract of insurance is one of perfect good faith (uferrimal fidei) not for
the insured alone, but equally so for the insurer, in fact, it is mere so for the latter, since its dominant
bargaining position carries with it stricter responsibility. We find no reversible error in the judgment
appealed from, wherefore the same is hereby affirmed. Costs against the appellant.

MANILA MAHOGANY MANUFACTURING CORP v. CA (ZENITH INSURANCE CORP)

FACTS
- From 6 March 1970 to 6 March 1971, MLA MAHOGANY insured its Mercedes Benz 4-door sedan with ZENITH.
- On 4 May 1970, the insured vehicle was bumped and damaged by a truck owned by San Miguel Corporation. For
the damage caused, ZENITH paid MLA MAHOGANY P5,000 in amicable settlement. MLA MAHOGANY's general
15
manager executed a Release of Claim, subrogating respondent company to all its right to action against San Miguel
Corporation.
- On 11 Dec 1972, ZENITH wrote Insurance Adjusters, Inc. to demand reimbursement from San Miguel. Insurance
Adjusters, Inc. refused reimbursement, alleging that San Miguel had already paid petitioner P4,500, as evidenced
by a cash voucher and a Release of Claim executed by the General Manager of petitioner.
- ZENITH thus demanded from petitioner reimbursement of the sum of P4,500 paid by San Miguel.

ISSUE
WON the insurer may recover the sum of P5,000
HELD
YES
Ratio Since the insurer can be subrogated to only such rights as the insured may have, should the insured, after
receiving payment from the insurer, release the wrongdoer who caused the loss, the insurer loses his rights against
the latter. But in such a case, the insurer will be entitled to recover from the insured whatever it has paid to the
latter, unless the release was made with the consent of the insurer.
Reasoning
- Although petitioner’s right to file a deficiency claim against San Miguel is with legal basis, without prejudice to the
insurer's right of subrogation, nevertheless, when Manila Mahogany executed another release claim discharging
San Miguel from "all actions, claims, demands and rights of action that now exist or hereafter arising out of or as a
consequence of the accident" after the insurer had paid the proceeds of the policy - the compromise agreement of
P5,000 being based on the insurance policy - the insurer is entitled to recover from the insured the amount of
insurance money paid. Since petitioner by its own acts released San Miguel, thereby defeating private respondent’s
right of subrogation, the right of action of petitioner against the insurer was also nullified.
- As held in Phil. Air Lines v. Heald Lumber Co., under Art. 2207, the real party in interest with regard to the
portion of the indemnity paid is the insurer and not the insured.

PAN MALAYAN INSURANCE CORPORATION VS COUR OF APPEALS G.R. NO. 81026 APRIL 03, 1990 (CASE
# 39)
FACTS
On December 10, 1985, PANMALAY filed a complaint for damages with the RTC of Makati against
private respondents Erlinda Fabie and her driver. PANMALAY averred the following: that it insured a
Mitsubishi Colt Lancer car with plate No. DDZ-431 and registered in the name of Canlubang
Automotive Resources Corporation [CANLUBANG]; that on May 26, 1985, due to the "carelessness,
recklessness, and imprudence" of the unknown driver of a pick-up with plate no. PCR-220, the insured
car was hit and suffered damages in the amount of P42,052.00; that PANMALAY defrayed the cost of
repair of the insured car and, therefore, was subrogated to the rights of CANLUBANG against the
driver of the pick-up and his employer, Erlinda Fabie; and that, despite repeated demands,
defendants, failed and refused to pay the claim of PANMALAY.
.
ISSUE: Whether petitioner insurance company has a cause of action against private respondent
RULING OF THE COURT

16
Yes. Article 2207 of the Civil Code is founded on the well-settled principle of subrogation. If the
insured property is destroyed or damaged through the fault or negligence of a party other than the
assured, then the insurer, upon payment to the assured, will be subrogated to the rights of the
assured to recover from the wrongdoer to the extent that the insurer has been obligated to pay.
Payment by the insurer to the assured operates as an equitable assignment to the former of all
remedies which the latter may have against the third party whose negligence or wrongful act caused
the loss. The right of subrogation is not dependent upon, nor does it grow out of, any privity of
contract or upon written assignment of claim. It accrues simply upon payment of the insurance claim
by the insurer

PHIL. AMERICAN GEN. INSURANCE CO, INC. vs. CA


G.R. No. 116940 June 11, 1997
FACTS:
On 6 July 1983 Coca-Cola Bottlers Philippines,
Inc., loaded on board "MV Asilda," a vessel owned and
operated by respondent Felman Shipping Lines, 7,500
cases of 1-liter Coca-Cola softdrink bottles to be
transported from Zamboanga City to Cebu City. The
shipment was insured with petitioner Philippine American
General Insurance Co., Inc., under Marine Open. On 7
July 1983, the vessel sank in the waters of Zamboanga
del Norte bringing down her entire cargo with her including
the subject 7,500 cases of 1-liter Coca-Cola softdrink
bottles.
on 29 November 1983 PHILAMGEN sued the
shipowner for sum of money and damages. On 15
February 1985 FELMAN filed a motion to dismiss based
on the affirmative defense that FELMAN had abandoned
all its rights, interests and ownership over "MV Asilda"
together with her freight and appurtenances for the
purpose of limiting and extinguishing its liability under Art.
587 of the Code of Commerce.

ISSUE:
WON the limited liability under Art. 587 of the
Code of Commerce should apply

DECISION:
Art. 587 of the Code of Commerce is NOT
applicable to the case at bar.
The ship agent is liable for
the negligent acts of the captain in the care of goods
17
loaded on the vessel. This liability however can be limited
through abandonment of the vessel, its equipment and
freightage as provided in Art. 587. Nonetheless, there are
exceptional circumstances wherein the ship agent could
still be held answerable despite the abandonment, as
where the loss or injury was due to the fault of the
shipowner and the captain.
The international rule is to the
effect that the right of abandonment of vessels, as a legal
limitation of a shipowner's liability, does not apply to cases
where the injury or average was occasioned by the
shipowner's own fault. It must be stressed at this point
that Art. 587 speaks only of situations where the fault or
negligence is committed solely by the captain. Where the
shipowner is likewise to be blamed, Art. 587 will not
apply.
It was already established at the outset that the
sinking of "MV Asilda" was due to its unseaworthiness
even at the time of its departure from the port of
Zamboanga. Closer supervision on the part of the
shipowner could have prevented this fatal miscalculation.
As such, FELMAN was equally negligent. It cannot
therefore escape liability through the expedient of filing a
notice of abandonment of the vessel by virtue of Art. 587
of the Code of Commerce.
WHEREFORE, the petition is GRANTED.
Respondent FELMAN SHIPPING LINES is ordered to pay
petitioner PHILIPPINE AMERICAN GENERAL
INSURANCE CO., INC

SVERIGES ANGFARTYGS ASSURANS FORENIN vs. GAN (1967).

DOCTRINE: [A]n insurer who pays the insured for loss or liability not covered by the policy
is not subrogated to the latter. However, even assuming that there was unwarranted — or
"volunteer" — payment, plaintiff could still recover what it paid — in effect — to the carrier
from defendant shipper under Art. 1236 of the Civil Code which allows a third person who
pays on behalf of another to recover from the latter, although there is no subrogation.

FACTS: Defendant Qua Chee Gan shipped on board the S.S. NAGARA, as per bills of
lading Exhs. A and B, 2,032,000 kilos of bulk copra at Siain, Quezon, consigned to DAL
International Trading Co., in Gdynia, Poland. The vessel first called at the port of
Karlshamn, Sweden, where it unloaded 969,419 kilos of bulk copra. Then, it proceeded to
18
Gdynia where it unloaded the remaining copra shipment. The actual outturn weights in the
latter port showed that only 1,569,429 kilos were discharged. Because of the alleged
confirmed cargo shortage, the Polish cargo insurers had to indemnify the consignee for the
value thereof. Thereafter, the former sued the shipowner, the Swedish East Asia Company,
in Gothenburg, Sweden. The latter, in turn sued, defendant and had it summoned to
Gothenburg. A settlement was effected between the Polish cargo insurers and the
shipowner. Plaintiff, as the indemnity insurer for the latter, paid approximately $60,733.53
to the Polish insurers. Claiming to have been subrogated to the rights of the carrier, plaintiff
sued defendant before the CFI of Manila to recover U.S. $60,733.53 plus 17% exchange
tax, with legal interest, as the value of the alleged cargo short shipment.
Defendant filed a motion to dismiss on the ground of prescription under the Carriage of
Goods by Sea Act.
ISSUE: WON plaintiff is subrogated to the rights of the carrier.
RULING: NO. [T]he rule is that an insurer who pays the insured for loss or liability not
covered by the policy is not subrogated to the latter. However, even assuming that there
was unwarranted — or "volunteer" — payment, plaintiff could still recover what it paid — in
effect — to the carrier from defendant shipper under Art. 1236 of the Civil Code which
allows a third person who pays on behalf of another to recover from the latter, although
there is no subrogation. But since the payment here was without the knowledge and
consent of defendant, plaintiff's right of recovery is defeasible by the former's defenses
since the Code is clear that the recovery is only up to the amount by which the defendant
was benefited.

PAUL FIRE & MARINE INSURANCE v MACONDRAY & CO

Facts:

-Winthrop Products, Inc., of New York shipped aboard the SS “Tai Ping”, owned and operated by
Wilhelm Wilhelmsen218 cartons and drums of drugs and medicine with Winthrop-Stearns Inc.,
Manila, Philippines as consingee. BarberSteamship Lines, Inc., agent of Wilhelm Wilhelmsen issued Bill
of Lading No. 34, in the name of Winthrop Products.

-The shipment was insured by the shipper against loss and/or damage with the St. Paul Fire & Marine
InsuranceCompany.

-“Tai Ping” arrived at the Port of Manila.

19
-The said shipment was discharged complete and in good order with the exception of one (1) drum
and several cartonswhich were in bad condition.

-Because consignee failed to receive the whole shipment and as several cartons of medicine were
received in badorder condition, Winthrop-Sterns Philippines filed the corresponding claim in the
amount of Pl,109.67 representingthe C.I.F. value of the damaged drum and cartons of medicine with
the carrier and the arrestre.

-However, both refused to pay.

-Winthrop-Sterns Philippines filed its claim with the insurer, St. Paul Fire & Marine insurance.

-The insurance company, on the basis of such claim, paid to the consignee the insured value of the lost
and damagedgoods, including other expenses in connection therewith, in the total amount
of $1,134.46.

-As subrogee of the rights of the shipper and/or consignee, the insurer, St. Paul Fire & Marine
Insurance Co., instituted with the Court of First Instance the present action against the defendants for
the recovery of said amount of $1,134.46, plus costs.

ISSUE:
WON the insurance carrier can recover the said amount claimed.

RULING: YES,
the plaintiff-appellant, as insurer, after paying the claim of the insured for
damages under the insurance, is subrogated merely to the rights of the assured. As
subrogee, it can recover only the amount that is recoverable by the latter. Since the right of
the assured, in case of loss or damage to the goods, is limited or restricted by the
provisions in the bill of lading, a suit by the insurer as subrogee necessarily is subject to
like limitations and restrictions

DOCTRINE:
Insurance carrier is subrogated merely to rights of the assured and can
recover from common carrier only the amount recoverable by the latter

FIREMAN’S FUND INSURANCE COMPANY AND THE FIRESTONE TIRE


AND RUBBER COMPANY OF THE PHILIPPINES v JAMILA & COMPANY
Flores

DOCTRINE:
When the insurance company pays for the loss, such payment operates as
an equitable assignment to the insurer of the property and all remedies which the insured
may have for the recovery thereof. That right is not dependent upon, nor does it grow out

20
of, any privity of contract, or upon written assignment of claim, and payment to the insured
makes the insurer an assignee in equity.

FACTS
Jamila/Veterans PH Scouts Security Agency contracted to supply security guards to
Firestone. Jamila assumed responsibility for the acts of its security guards.
First Quezon City Insurance executed a bond for P20K to guarantee Jamila’s
obligation under that contract.
Later on Firestone’s properties valued at P11,925 were allegedly lost due to the acts
of its employees who connived with Jamila’s security guard
Fireman’s Fund, as insurer paid to Firestone the amount of the loss and Fireman’s
Fund was subrogated to Firestone’s right to get reimbursement from Jamila. Jamila
and its surety (First Quezon Insurance) failed to pay the amount of the loss.
Fireman invoked Article 2207, as the insurer Fireman’s Fund is entitled to go after the
person or entity that violated its contractual commitment to answer for the loss insured
against.
Jamila contends that it did not consent to the subrogation of Fireman’s Fund to
Firestone’s right to get reimbursement from Jamila and its surety. Further arguing that
legal subrogation under Art 2207 of the Civil Code requires the debtor’s consent and
that according to Art 1302, the instant case is not among the cases mentioned when
legal subrogation can take place.

ISSUE:
WON Fireman’s Fund can subrogate to the rights of Jamilia?

RULING:
YES!
Article 2207 is a restatement of a settled principle of American
jurisprudence. Subrogation has been referred to as the doctrine of substitution. It „is an
arm of equity that may guide or even force one to pay a debt for which an obligation was
incurred but which was in whole or in part paid by another.
Subrogation is founded on principles of justice and equity, and its operation is governed by
principles of equity. It rests on the principle that substantial justice should be attained
regardless of form, that is, its basis is the doing of complete, essential, and perfect justice
between all the parties without regard to form.
Subrogation is a normal incident of indemnity insurance. Upon payment of the loss, the
insurer is entitled to be subrogated
pro tanto
to any right of action which the insured may
have against the third person whose negligence or wrongful act caused the loss
When the insurance company pays for the loss, such payment operates as an equitable
assignment to the insurer of the property and all remedies which the insured may have for
the recovery thereof. That right is not dependent upon, nor does it grow out of, any privity
21
of contract, or upon written assignment of claim, and payment to the insured makes the
insurer an assignee in equity

F.F. Cruz v CA – Dennis Reyes

DOCTRINE
: in Article 2207. Upon payment of the loss incurred by the insured, the insurer
is entitled to be subrogated pro tanto to any right of action which the insured may have
against the third person whose negligence or wrongful act caused the loss.

FACTS
: The furniture manufacturing shop of petitioner was situated adjacent to the
residence of private respondents. Private respondent, Gregorio Mable, first approached
the petitioner's plant manager to request that a firewall be constructed between the shop
and private respondents' residence. The request was repeated several times but they fell
on deaf ears.
A fire broke out in petitioner's shop. Petitioner's employees tried to put out the fire, but their
efforts proved futile. The fire spread to private respondents' house. Both the shop and the
house were razed to the ground. The cause of the conflagration was never discovered.
Private respondents collected P35,000.00 on the insurance on their house and the
contents thereof.
Private respondents filed an action for damages against petitioner. Trial Court awarded
damages in favor of the private respondent; Court of Appeals: affirmed the decision of the
trial court

ISSUE
: Even though the Respondent received a sum of money (P35,000) from the insurer,
can the latter collect from the petitioner

RULING
: The Court finds that petitioner is liable for damages to private respondents, the
fact that private respondents have been indemnified by their insurer in the amount of
P35,000.00 for the damage caused to their house and its contents has not escaped the
attention of the Court. Hence, the Court holds that in accordance with Article 2207 of the
Civil Code the amount of P35,000.00 should be deducted from the amount awarded as
damages. Said article provides:
Art. 2207. Xxx If the amount paid by the insurance company does not fully cover the
injury or loss, the aggrieved party shall be entitled to recover the deficiency from the
person causing the loss or injury.

RIZAL SURETY v. MANILA RAILROAD COMPANY (1968)

22
DOCTRINE:
Insurer after paying the claim of the insured for damages under the insurance
is subrogated merely to the rights of the insured and therefore can necessarily recover only
that to what was recoverable by the insured.

FACTS:
On or about November 29, 1960, the vessel, SS Flying trader, loaded on board at
Genoa, Italy for shipment to Manila, among other cargoes, 6 cases OMH Special Single
Colour Offset Press machine, for which Bill of Lading No. 1 was issued, consigned to
Suter, Inc.
On or about January 16, 1961, the vessel arrived in Manila and subsequently discharged
complete and in good order the aforementioned shipment into the custody of defendant
Manila Port Service as arrastre operator.
While one of the six cases was being lifted and loaded by the crane of Manila Port Service
into the consignee’s truck, it was dropped by the crane and as a consequence the machine
was heavily damaged.
The plaintiff, as the insurer, paid the consignee the amount of P16,500 representing
damages by way of costs of replacement parts and repairs to put the machine on a
working condition. The plaintiff also paid P16,680.70 to the International Adjustment
Bureau as adjuster’s fee.
The paragraph 15 of the management contract between the Bureau of Customs and
Manila Port Service states that the Company’s liability is limited to P500.00 per package
unless the value of the goods is otherwise, specified, declared or manifested and the
corresponding arrastre has been paid.

ISSUE:
Whether or not the insurance company can collect more than what was stipulated
in the Management Contract?

RULING:
No. The literal language of Article 2207, however, does not warrant such an
interpretation. It is there made clear that in the event that the property has been insured
and the Insurance Company has paid the indemnity for the injury or loss sustained, it "shall
be subrogated to the rights of the insured against the wrong-doer or the person who has
violated the contract."
Plaintiff-appellant Insurance Company, therefore, cannot recover from defendants an
amount greater than that to which the consignee could lawfully lay claim. The management
contract is clear. The amount is limited to Five Hundred Pesos (P500.00).
Plaintiff-appellant Rizal Surety and Insurance Company, having been subrogated merely to
the rights of the consignee, its recovery necessarily should be limited to what was
recoverable by the insured. The lower court therefore did not err when in the decision
appealed from, it limited the amount which defendants were jointly and severally to pay
plaintiff-appellants to "Five Hundred Pesos (P500.00) with legal interest thereon from
23
January 31, 1962, the date of the filing of the complaint, . . . ."

WHITE GOLD MARINE SERVICES ,INC. VS. PIONEER INSURANCEAND SURETY CORPORATION AND THE
STEAMSHIP MUTUAL UNDERWRITING ASSOCIATION G.R. No. L-44059, 28 October 1977
FACTS: White Gold Marine Services, Inc. procured a protection and indemnity coverage for its vessels
from the Steamship Mutual Underwriting Association Limited through Pioneer Insurance and Surety
Corporation. White Gold was issued a certificate of entry and acceptance. Pioneer also issued receipts
evidencing payments for the coverage. When White Gold failed to fully pay its accounts, Steamship
Mutual refused to renew the coverage. Steamship Mutual thereafter filed case against White Gold for
collection of sum of money to recover the latters unpaid balance. White Gold on the other hand, filed
a complaint before the Insurance Commission claiming that Steamship Mutual Violated Sections 186
and 187, while Pioneer violated Sections 299 to 301 of the Insurance Code.
ISSUE: Whether or not Steamship Mutual, a Protection and Insurance Club, engaged in the business in
the Philippines.
HELD: The test to determine if a contract is an insurance contract or not, depends on the nature of the
promise, the act required to be performed, and the exact nature of the agreement in the light of the
occurrence, contingency, or circumstances under which the performance becomes requisite. It is not
by what it is called. Basically, an insurance contract is a contract of indemnity. In it, one undertakes for
a consideration to indemnify another against loss, damage or liability arising from unknown or
contingent event. In particular, a marine insurance undertakes to indemnify the assured against
marine losses, such as the losses incident to a marine adventure. Section 99 of the Insurance Code
enumerates the coverage of marine insurance. By definition then, Steamship Mutual as a P and I Club
is a mutual insurance association engaged in the marine insurance business.

THE MANUFACTURERS LIFE INSURANCE CO vs. BIBIANO L. MEER, in the capacity as Collector of
Internal Revenue
FACTS:
Manufacturer Life Insurance Company is a corporation duly organized in Canada with head office at
Toronto. It is duly registered and licensed to engage in life insurance business in the Philippines, and
maintains a branch office in Manila. It was engaged in such business in the Philippines for more than
five years before and including the year 1941. But due to the exigencies of the war it closed the
branch office at Manila during 1942 up to September 1945. In the course of its operations before the
war, plaintiff issued a number of life insurance policies in the Philippines containing stipulations
referred to as non-forfeiture clauses. Since the insured failed to pay from 1942 to 1946, the company
applied the provision of the automatic premium loan clauses; and the net amount of premiums so
24
advanced or loaned totaled P1,069,254.98. On this sum the defendant Collector of Internal Revenue
assessed P17,917.12. The assessment was made pursuant to section
255 of the NIRC which put taxes on insurance premiums paid by money, notes, credits or any
substitutes for money.
Manufacturer contended that when it made premium loans or premium advances by virtue of the
non-forfeiture clauses, it did not collect premiums within the meaning of the above sections of the
law, and therefore it is not amendable to the tax provided.
ISSUE/S:
(a) Whether or not premium advances made by plaintiffappellant under the automatic premium loan
clause of its policies are "premium collected" by the Company subject to tax (b) Whether or not, in the
application of the automatic premium loan clause of plaintiff-appellant's policies, there is "payment in
money, notes, credit, or any substitutes for money
RULING OF THE COURT: BOTH IN AFFIRMATIVE.
RATIONALE OF THE COURT BY CITING AN INSTANCE: 1. “A person secures a 20-years endowment
policy for P5,000 from Manufacturers and pays an annual premium of P250. He pays the first ten
yearly premiums amounting to P2,500 and on this amount plaintiffappellant pays the taxes. Also, the
cash value of said policy after the payment of the 10th annual premium amounts to P1,000." When on
the eleventh year the annual premium fell due and the insured remitted no money within the grace
period, the insurer treated the premium then overdue as paid from the cash value, the amount being
loan to the policyholder who could discharge it at anytime with interest at 6 per cent. The insurance
contract, therefore, continued in force for the eleventh year.”
Under the circumstances described, did the insurer collect the amount of P250 as the annual premium
for the eleventh year on the said policy? In effect the Manufacturers Life Insurance Co. loaned to the
person P250 and the latter in turn paid with that sum the annual premium on his policy. The Company
therefore collected the premium for the eleventh year.
"How could there be such a collection when insurer becomes a creditor, acquires a lien on the policy
and is entitled to collect interest on the amount of the unpaid premiums?".
Wittingly, the "premium" and the "loan" have been interchanged in the argument. The insurer
"became a creditor" of the loan, but not of the premium that had already been paid. And it is entitled
to collect interest on the loan, not on the premium.
The insured paid the premium for the eleventh; but in turn he became a debtor of the company for
the sum of P250. This debt he could repay either by later remitting the money to the insurer or by
letting the cash
value compensate for it. The debt may also be deducted from the amount of the policy should he die
thereafter during the continuance of the policy.

25
There was new credit for the advances made. True, the company could not sue the insured to enforce
that credit. But it has means of satisfaction out of the cash surrender value.
Here again it may be urged that if the credit is paid out of the cash surrender value, there were no
new funds added to the company's assets. Cash surrender value "as applied to life insurance policy, is
the amount of money the company agrees to pay to the holder of the policy if he surrenders it and
releases his claims upon it. The more premiums the insured has paid the greater will be the surrender
value; but the surrender value is always a lesser sum than the total amount of premiums paid."
The cash value or cash surrender value is therefore an amount which the insurance company holds in
trust for the insured to be delivered to him upon demand. It is therefore a liability of the company to
the insured. Now then, when the company's credit for advances is paid out of the cash value or cash
surrender value, that value and the company's liability is thereby dismissed. Consequently, the net
assets of the insurance company increase.
2. The insurer agreed to consider the premium paid on the strength of the automatic loan. The
premium was paid by means of a "note" or "credit" or "other substitute for money" and the taxes due
because section 255 above quoted levies taxes according to the total premiums collected by the
insurer "whether such premiums are paid in money, notes, credits or any substitutes for money.

Prudential Guarantee Assurance Inc. vs. Trans Asia Shipping Lines


G.R. No. 151890
June 20, 2006
Facts:
Trans Asia is the owner of the vessel M/V Asia Korea. Prudential Guarantee and Assurance Inc.
insured said vessel for loss/damage of the hull and machinery arising from perils of fire and
explosion beginning from the period of July 1, 1993 until July 1, 1994. While the policy was in
force, a fire broke out. Trans Asia file its notice of claim for damages sustained by the vessel. It
also reserved its right to subsequently notify Prudential as to the full amount of the claim upon
final survey and determination by the average adjuster Richard Hogg International of the damage
sustained by the reason of fire. Trans Asia executed a document denominated "Loan and Trust
Receipt" amounting to Php 3,000,000. Prudential Guarantee and Assurance Inc. denied the
former's claim and requested for the return of the said amount. The insurance company contends
that there was a breach in the policy conditions, specifically, "Warranted Vessel Classed and
Class Maintained".

Issue/s:
1.) WON there was a breach in the warranty of the contract.
Held:
The Supreme Court held that:
1.) Prudential failed to establish that Trans Asia had violated and breached the policy condition
provided in the insurance contract. The latter was able to establish proof of loss and coverage of
26
the loss. Prudential also made a categorical admission at the time of the procurement of the
insurance contract that the vessel was properly classified by the Bureau Veritas.
Assuming that there was a breach in the policy, the renewal of the insurance policy for two
consecutive years after the loss is deemed as a waiver on the part of Prudential. Breach of a
warranty or of a condition renders the contract defeasible at the option of the insurer; but if he so
elects, he may waive his privilege and power to rescind by the mere expression of an intention so
to do.

FILIPINAS COMPANIA DE SEGUROS vs. CHRISTERN HUENEFELD AND CO.

[G.R. No. L-2294; May 25, 1951]

FACTS:

 October 1, 1941: Christern Huenefeld and co., inc. (Christern), a company whose major
stockholders are German, paid P1M and obtained a fire policy from Filipinas Cia. de Seguros
(Filipinas)
 December 10, 1941: S. declared a war against Germany
 February 27, 1942 (during the japanese occupation): the building and insured merchandise
were burned
o their claimed from Filipinas and the salvage goods were auctioned for P92,650 who
refused since Christen was organized under the Philippine laws, it was under American
jurisdiction which is an enemy of the Germans
 April 9, 1943: The Director of Bureau of Financing ordered Filipinas to pay the P92,650 to
Christen and it did.

Issue:

Whether or not Filipinas is liable to Christern, Huenfeld & Co.

Held:

NO.

Majority of the stockholders of Christern were German subjects. This being so, SC ruled that said
corporation became an enemy corporation upon the war between the US and Germany. The Phil
Insurance Law in Sec. 8 provides that anyone except a public enemy may be insured. It stands to
reason that an insurance policy ceases to be allowable as soon as an insured becomes a public enemy.

27
The purpose of the war is to cripple the power ad exhaust the resources of the enemy, and it is
inconsistent that one country should destroy its enemy property and repay in insurance the value of
what has been so destroyed, or that it should in such manner increase the resources of the enemy or
render it aid.

All individuals who compose the belligerent powers, exist as to each other, in a state of utter exclusion
and are public enemies. Christern having become an enemy corporation on Dec. 10. 1941, the
insurance policy issued in his favor on Oct. 1, 1941 by Filipinas had ceased to be valid and enforceable,
and since the insured goods were burned after Dec. 10, 1941, and during the war, Christern was NOT
entitled to any indemnity under said policy from Filipinas.

Elementary rules of justice require that the premium paid by Christern for the period covered by the
policy from Dec. 10, 1941 should be returned by Filipinas.

CONSTANTINO V. ASIA LIFE-


Facts: > Appeal consolidates two cases. > Asia life insurance Company (ALIC) was incorporated in
Delaware. > For the sum of 175.04 as annual premium duly paid to ALIC, it issued Policy No. 93912
whereby it insured the life of Arcadio Constantino for 20 years for P3T with Paz Constantino as
beneficiary.  First premium covered the period up to Sept. 26, 1942. No further premiums were paid
after the first premium and Arcadio died on Sept. 22, 1944.
25

> Due to Jap occupation, ALIC closed its branch office in Manila from Jan. 2 1942-1945. > On Aug. 1,
1938, ALIC issued Policy no. 78145 covering the lives of Spouses Tomas Ruiz and Agustina Peralta for
the sum of P3T for 20 years. The annual premium stipulated was regularly paid from Aug. 1, 1938 up
to and including Sept. 30, 1940.  Effective Aug. 1, 1941, the mode of payment was changed from
annually to quarterly and such quarterly premiums were paid until Nov. 18, 1941.  Last payment
covered the period until Jan. 31, 1942.  Tomas Ruiz died on Feb. 16, 1945 with Agustina Peralta as his
beneficiary. > Due to Jap occupation, it became impossible and illegal for the insured to deal with
ALIC. Aside from this the insured borrowed from the policy P234.00 such that the cash surrender
value of the policy was sufficient to maintain the policy in force only up to Sept. 7, 1942. > Both
policies contained this provision: All premiums are due in advance and any unpunctuality in making
such payment shall cause this policy to lapse unless and except as kept in force by the grace period
condition. > Paz Constantino and Agustina Peralta claim as beneficiaries, that they are entitled to
receive the proceeds of the policies less all sums due for premiums in arrears. They also allege that
28
non-payment of the premiums were caused by the closing of ALIC’s offices during the war and the
impossible circumstances by the war, therefore, they should be excused and the policies should not be
forfeited. > Lower court ruled in favor of ALIC.

Issue:

May a beneficiary in a life insurance policy recover the amount thereof although the insured died after
repeatedly failing to pay the stipulated premiums, such failure being caused by war?

Held: NO. Due to the express terms of the policy, non-payment of the premium produces its
avoidance. In Glaraga v. Sun Life, it was held that a life policy was avoided because the premium had
not been paid within the time fixed; since by its express terms, non-payment of any premium when
due or within the 31 day grace period ipso fact caused the policy to lapse.

When the life insurance policy provides that non-payment of premiums will cause its forfeiture, war
does NOT excuse nonpayment and does not avoid forfeiture. Essentially, the reason why punctual
payments are important is that the insurer calculates on the basis of the prompt payments.
Otherwise, malulugi sila.

It should be noted that the parties contracted not only as to peace time conditions but also as to war-
time conditions since the policies contained provisions applicable expressly to wartime days. The
logical inference therefore is that the parties contemplated the uninterrupted operation of the
contract even if armed conflict should ensue.

VICTORIA HIDALGO VDA. DE CARRERO vs. MANUFACTURERS LIFE INSURANCE CO.

Insurance Contract in Times of War – US Rule (Statham Rule)

FACTS:

In February 1934, Juan Carrero was issued an insurance policy by Manufacturer’s Life Insurance Co.
(Manulife). Carrero incurred a $5,500.00 loan from Manulife in 1941. However, during the Japanese
occupation from 1942 to 1944, Carrero was not able to pay his premium to Manulife. In February
1945, Carrero died. His widow Victoria Hidalgo filed an insurance claim but Manulife refused due to
Carrero’s nonpayment.

29
ISSUE:

Whether or not Hidalgo’s insurance policy has been forfeited considering that nonpayment was due to
the war.

HELD:

Yes. The case of Constantino vs Asia Life has been reiterated in this case. It must be conceded that
promptness of payment is essential in the business of life insurance. It is on this basis that insurance
companies are enabled to offer assurance at the favorable rates they do. Forfeiture for non-payment is
a necessary means of protecting themselves from embarrassment. Unless it were enforceable, the
business would be thrown into utter confusion.

This case, therefore, is one in which time is material and of the essence of the contract. Non-payment
at the day involves absolute forfeiture if such be the terms of the contract, as is the case here. Courts
cannot with safety vary the stipulation of the parties by introducing equities for the relief of the
insured against their own negligence.

The United States rule (Statham case) declares that the contract is not merely suspended, but is
abrogated by reason of non-payment of premium, since the time of the payment is peculiarly of the
essence of the contract. It additionally holds that it would be unjust to allow the insurer to retain the
reserve value of the policy, which is the excess of the premiums paid over the actual risk carried
during the years when the policy had been in force.

GEAGONIA vs. CA

[G.R. No. 114427 February 6, 1995]

Facts:

Geagonia, owner of a store, obtained from Country Bankers fire insurance policy for P100,000.00. The
1 year policy and covered the stock trading of dry goods.

The policy noted the requirement that

“3. The insured shall give notice to the Company of any insurance or insurances already effected, or
which may subsequently be effected, covering any of the property or properties consisting of stocks in
trade, goods in process and/or inventories only hereby insured, and unless notice be given and the
particulars of such insurance or insurances be stated therein or endorsed in this policy pursuant to
Section 50 of the Insurance Code, by or on behalf of the Company before the occurrence of any loss or
damage, all benefits under this policy shall be deemed forfeited, provided however, that this condition
shall not apply when the total insurance or insurances in force at the time of the loss or damage is not
more than P200,000.00.”

30
The petitioners’ stocks were destroyed by fire. He then filed a claim which was subsequently denied
because the petitioner’s stocks were covered by two other fire insurance policies for Php 200,000
issued by PFIC. The basis of the private respondent’s denial was the petitioner’s alleged violation of
Condition 3 of the policy.

Geagonia then filed a complaint against the private respondent in the Insurance Commission for the
recovery of P100,000.00 under fire insurance policy and damages. He claimed that he knew the
existence of the other two policies. But, he said that he had no knowledge of the provision in the
private respondent’s policy requiring him to inform it of the prior policies and this requirement was
not mentioned to him by the private respondent’s agent.

Issues:

1. WON the petitioner had not disclosed the two insurance policies when he obtained the fire
insurance and thereby violated Condition 3 of the policy.
2. WON he is prohibited from recovering

Held:

1. Yes.
2. No.

Petition Granted

Ratio:

1. The court agreed with the CA that the petitioner knew of the prior policies issued by the PFIC.
His letter of 18 January 1991 to the private respondent conclusively proves this knowledge. His
testimony to the contrary before the Insurance Commissioner and which the latter relied upon
cannot prevail over a written admission made ante litem motam. It was, indeed, incredible that
he did not know about the prior policies since these policies were not new or original.
2. Stated differently, provisions, conditions or exceptions in policies which tend to work a
forfeiture of insurance policies should be construed most strictly against those for whose
benefits they are inserted, and most favorably toward those against whom they are intended to
operate.

With these principles in mind, Condition 3 of the subject policy is not totally free from ambiguity and
must be meticulously analyzed. Such analysis leads us to conclude that (a) the prohibition applies only
to double insurance, and (b) the nullity of the policy shall only be to the extent exceeding P200,000.00
of the total policies obtained.

Furthermore, by stating within Condition 3 itself that such condition shall not apply if the total
insurance in force at the time of loss does not exceed P200,000.00, the private respondent was
amenable to assume a co-insurer’s liability up to a loss not exceeding P200,000.00. What it had in

31
mind was to discourage over-insurance. Indeed, the rationale behind the incorporation of “other
insurance” clause in fire policies is to prevent over-insurance and thus avert the perpetration of fraud.
When a property owner obtains insurance policies from two or more insurers in a total amount that
exceeds the property’s value, the insured may have an inducement to destroy the property for the
purpose of collecting the insurance. The public as well as the insurer is interested in preventing a
situation in which a fire would be profitable to the insured.

Great Pacific Life vs. CA gr 11389

INSURANCE LAW: Parties in Insurance Contract

FACTS:

Great Pacific Life Assurance Corporation (Grepalife) executed a contract of group life insurance with
Development Bank of the Philippines (DBP) wherein Grepalife agreed to insure the lives of eligible
housing loan mortgagors of DBP.

One such loan mortgagor is Dr. Wilfredo Leuterio. In an application form, Dr. Leuterio answered
questions concerning his test, attesting among others that he does not have any heart conditions and
that he is in good health to the best of his knowledge.

However, after about a year, Dr. Leuterio died due to “massive cerebral hemorrhage.” When DBP
submitted a death claim to Grepalife, the latter denied the claim, alleging that Dr. Leuterio did not
disclose he had been suffering from hypertension, which caused his death. Allegedly, such non-
disclosure constituted concealment that justified the denial of the claim.

Hence, the widow of the late Dr. Leuterio filed a complaint against Grepalife for “Specific Performance
with Damages.” Both the trial court and the Court of Appeals found in favor of the widow and ordered
Grepalife to pay DBP.

ISSUE:

 Whether the CA erred in holding Grepalife liable to DBP as beneficiary in a group life insurance
contract from a complaint filed by the widow of the decedent/mortgagor

HELD:

The rationale of a group of insurance policy of mortgagors, otherwise known as the “mortgage
redemption insurance,” is a device for the protection of both the mortgagee and the mortgagor. On
the part of the mortgagee, it has to enter into such form of contract so that in the event of the
32
unexpected demise of the mortgagor during the subsistence of the mortgage contract, the proceeds
from such insurance will be applied to the payment of the mortgage debt, thereby relieving the heirs
of the mortgagor from paying the obligation. In a similar vein, ample protection is given to the
mortgagor under such a concept so that in the event of death, the mortgage obligation will be
extinguished by the application of the insurance proceeds to the mortgage indebtedness. In this type
of policy insurance, the mortgagee is simply an appointee of the insurance fund. Such loss-payable
clause does not make the mortgagee a party to the contract.

The insured, being the person with whom the contract was made, is primarily the proper person to
bring suit thereon. Subject to some exceptions, insured may thus sue, although the policy is taken
wholly or in part for the benefit of another person, such as a mortgagee.

And since a policy of insurance upon life or health may pass by transfer, will or succession to any
person, whether he has an insurable interest or not, and such person may recover it whatever the
insured might have recovered, the widow of the decedent Dr. Leuterio may file the suit against the
insurer, Grepalife.

EMILIO GONZALES LAO v. THE YEK TONG LIN FIRE AND MARINE INSURANCE CO., LTD.
55 Phil. 386, December 13, 1930

Facts: This case involved an action to recover from defendant, The Yek Tong Lin Fire and
Marine Insurance Co., Ltd., the two insurance policies totaling P100,000 upon leaf tobacco. On
January 11, 1928, a fire destroyed a building on Soler Street No. 188 damaging the leaf tobacco
stored inside. Plaintiff Emilio Gonzales La O, the owner of the leaf tobacco, filed insurance
claims against three insurance companies including the P100,000 from defendant. The lower
court sentenced the defendant to pay the plaintiff the amount P100,000 and legal interest plus
costs. The case reached the Supreme Court where the defendant made several assignments of
error which included the plaintiff’s failure to notify the defendant corporation in writing of the
other insurance policies thereby violating Article 3 of the condition of the policies in question.
Thus, according to the defendant, the policies are null and void.

Issue: W hether or not the insurance policies should be rendered null and void for violating
Article 3 of the said policies because of the plaintiff failed to notify the defendant company in
writing of the other insurance policies.

Ruling: N o. The defendant’s answer showed that it had knowledge of the existence of other
policies obtained by the plaintiff from other insurance companies. By way of special defense,
the fact that there exist other policies issued by the companies was mentioned therein. If, with
the knowledge of existence of other insurances which the defendant deemed violations of the
contract, it has preferred to continue the policy, its action amounts to a waiver of the annulment
of the contract, in accordance with the following doctrine in 19 Cyc., 791, 792:.

33
FAILURE TO ASSERT FORFEITURE — IN GENERAL. — While the weight of authority
is that a policy conditioned to become void upon a breach of a warranty is void ipso facto
upon such a breach without formal proceedings on the part of the insurer, yet it is true
that such conditions are inserted for the benefit of the insurer and may be waived, and
that the insurer may elect to continue the policy despite the breach. If it does the policy is
revived and restored. Its failure to assert a forfeiture therefore is at least evidence
tending to show a waiver thereof. Many authorities go further, however, and hold that the
failure to assert a forfeiture after knowledge of a ground thereof will amount of itself to
waiver.
The following clause has been inserted with a typewriter in the policies: "Subject to clauses G
and A and other insurances with a special short period attached to this policy." And attached to
said policies issued by the defendant there is a sheet of "Other insurances" with the amount and
the assurance companies in blank, which, according to the plaintiff, constitutes a notification that
there were other insurances existing at the time. Furthermore, the appellant cannot invoke the
violation of article 3 of the conditions of the insurance policies for the first time on appeal, having
failed to do so in its answer; besides, as the appellee correctly contends in his brief, Guillermo
Cu Unjieng, who was then president and majority shareholder of the defendant company, the Yek
Tong Lin Fire & Marine Insurance Co., knew that there were other insurances.

Bonifacio Bros., Inc., et al. vs Enrique Mora, et al.


GR No. L-20853, May 29, 1967
Facts:
Enrique Mora, owner of an Oldsman sedan mortgaged the same to the HS Reyes Inc. Thereafter, the
automobile was insured with the State Bonding & Insurance Co., Inc with the provision that ‘Loss, if
any, is payable to HS Reyes Inc.’ by virtue of the fact that said Oldsman sedan was mortgaged in favor
of the latter.
During the effectivity of the insurance contract, the car met with an accident. Enrique Mora, without
the knowledge and consent of HS Reyes Inc. authorized Bonifacio Bros. Inc. to furnish the labor and
materials and some of which were supplied by the Ayala Auto Parts Co. Proceeds of the insurance
policy was not given to Bonifacio Bros. Inc. and Ayala Auto Parts Co., hence, complaint was filed
before Municipal Court of Manila against Enrique Mora and the insurance company for the labor and
materials supplied.
The appellants argued that they are privy to the contract. On the other hand, the insurance company
maintains that appellants are not mentioned in the contract as parties thereto nor is there any clause
or provision from which it can be inferred that there is an obligation on the part of the insurance
company to pay the cost of repairs directly to them.

34
Issue: Whether there is privity of contract between the Bonifacio Bos. Inc and the Ayala Auto Parts Co.
on the one hand and the insurance company on the other.
Ruling:
The appellants are not privy to the contract, hence, they have no right of action against the insurance
company.
A policy of insurance is a distinct and independent contract between the insured and insurer, and third
persons have no right either in a court of equity, or in a court of law, to the proceeds of it, unless there
be some contract of trust, expressed or implied, by the insured and the third person.
It is fundamental that contracts take effect only between the parties thereto, except on some specific
cases provided by law where the contract contains some stipulation in favor of a third person (Art.
1311, Civil Code) Such stipulation is known as stipulation pour autrui or a provision in favor of a third
person not party to the contract. In the instant case, the insurance contract does not contain any
words or clauses to disclose an intent to give any benefit to any repairmen or material men in
case of repair of the car in question. The “loss payable” clause of the insurance policy stipulates that
“Loss, if any, is payable to HS Reyes, Inc” indicating that it was only the HS Reyes Inc. which they
intended to benefit.

VIOLETA LALICAN vs. THE INSULAR LIFE ASSURANCE COMPANY LIMITED G.R. No. 183526, August 25,
2009, 597 SCRA 159
FACTS: Eulogio, the husband of herein petitioner, applied for an insurance policy the value of which is
P1,500,000.00. Under the policy terms, Eulogio is obliged to pay the premiums on a quarterly basis,
until the end of the 20-year period of the policy. It was likewise stated therein that the insured has 31-
day grace period for the payment of each premium subsequent to the first and that default in any
payment of said premiums shall result in the automatic lapse of the said policy. Eulogio failed to pay a
premium even after the lapse of the 31-day grace period. Hence, the policy lapsed and became void.
He filed an Application for Reinstatement of said policy and paying the amount of the premium due.
However, Insular Life notified him that they could not fully process his application because the amount
he paid is inadequate to cover the accrued interests. Hence, he again applied for the reinstatement of
said policy this time, together with the required amount. The husband of the insurance agent was the
one who received his application because the agent was away at that time. Within the same day, the
insured died. This fact was unknown to the agent who then submitted Eulogio’s application for
reinstatement to the Insular Life Regional Office. Violeta then filed a claim for payment of the full
proceeds of the policy. However, the company said that she is not entitled to the insurance proceeds
because they claimed that the policy
was not reinstated during her husband’s lifetime and good health.
ISSUE: Whether or not Eulogio was able to reinstate the lapsed insurance policy before his death
35
HELD: NO. The Court agrees with the RTC that the conditions for reinstatement under the Policy
Contract and Application for Reinstatement were written in clear and simple language, which could
not admit of any meaning or interpretation other than those that they so obviously embody. Violeta
did not adduce any evidence that Eulogio might have failed to fully understand the import and
meaning of the provisions of his Policy Contract and/or Application for Reinstatement both of which
he voluntarily signed. While it is a cardinal principle of insurance law that a policy or contract of
insurance is to be construed liberally in favor of the insured and strictly as against the insurer
company, yet, contracts of insurance, like other contracts are to be construed according to the sense
and meaning of the terms, which the parties themselves have used, if such terms are clear and
unambiguous, they must be taken and understood in their plain, ordinary and popular sense.
WHEREFORE, premises considered, the Court DENIES the instant Petition for Review on Certiorari
under Rule 45 of the Rules of Court. The Court AFFIRMS the Orders dated 10 April 2008 and 3 July
2008 of the RTC of Gapan City, Branch 34, in Civil Case No. 2177, denying petitioner Violeta R. Lalican’s
Notice of Appeal, on the ground that the Decision dated 30 August 2007 subject thereof, was already
final and executor. No costs.

THE INSULAR LIFE ASSURANCE COMPANY, LTD. Vs. CARPONIA T. EBRADO G.R. No. L-44059, 28 October
1977 80 SCRA 181
FACTS: Buenaventura Ebrado was issued by petitioner company an insurance policy wherein he
designated Carponia, his commonlaw wife, as his revocable beneficiary. When Buenaventura died,
Carponia filed with the insurer a claim for the proceeds of the policy in the total amount of
P11,745.73. Pascuala Vda. de Ebrado, Buenaventura’s legal wife, likewise filed her claim as the widow
of the deceased insured. She asserts that she is the one entitled to the insurance proceeds, not the
common-law wife, Carponia.
ISSUE: Whether or not a common-law wife named as beneficiary in the life insurance policy of a
legally married man can claim the proceeds thereof in case of death of the latter
HELD: NO. When not otherwise specifically provided for by the Insurance Law, the contract of life
insurance is governed by the general rules of the civil law regulating contracts. And under Article 2012
of the same Code, “any person who is forbidden from receiving any donation under Article 739 cannot
be named beneficiary of a life insurance policy by the person who cannot make a donation to him.
Common-law spouses are, definitely, barred from receiving donations from each other. In essence, a
life insurance policy is no different from a civil donation insofar as the beneficiary is concerned. Both
are founded upon the same consideration: liberality. A beneficiary is like a donee, because from the
premiums of the policy which the insured pays out of liberality, the beneficiary will receive the
proceeds or profits of said insurance. As a consequence, the proscription in Article 739 of the new
Civil Code should equally operate in life insurance contracts. The mandate of Article 2012 cannot be
laid aside: any person who cannot receive a donation cannot be named as beneficiary in the life
insurance policy of the person who cannot make the donation. Under American law, a policy of life
36
insurance is considered as a testament and in construing it, the courts will, so far as possible treat it as
a will
and determine the effect of a clause designating the beneficiary by rules under which wins are
interpreted. ACCORDINGLY, the appealed judgment of the lower court is hereby affirmed. Carponia T.
Ebrado is hereby declared disqualified to be the beneficiary of the late Buenaventura C. Ebrado in his
life insurance policy. As a consequence, the proceeds of the policy are hereby held payable to the
estate of the deceased insured. Costs against Carponia T. Ebrado

El Oriente, Fabrica de Tabacos, Inc., v. Posadas (1931)


G.R. No. 34774 September 21, 1931
Lessons Applicable: Pecuniary Interest (Insurance)

FACTS:

 March 18, 1925: El Oriente, Fabrica de Tabacos, Inc. in order to protect itself against the loss
that it might suffer by reason of the death of its manager, A. Velhagen, who had more than 35
years of experience in the manufacture of cigars in the Philippine Islands, and whose death
would be a serious loss procured from the Manufacturers Life Insurance Co., of Toronto,
Canada, thru its local agent E.E. Elser, an insurance policy on the life of A. Velhagen for $50,000
o designated itself as the sole beneficiary

 Upon the death of A. Velhagen in the year 1929, El Oriente received all the proceeds of the life
insurance policy, together with the interests and the dividends accruing thereon, aggregating
P104,957.88
 Collector of Internal Revenue assessed and levied the sum of P3,148.74 as income tax on the
proceeds of the insurance policy which tax El Oriente paid

ISSUE: W/N proceeds of life insurance policies paid to corporate beneficiaries upon the death of the
insured are also exempted

HELD: YES. reversed and favoring El Oriente


 In reality, what the plaintiff received was in the nature of an indemnity for the loss which it
actually suffered because of the death of its manager and not taxable income

GERCIO vs. SUN LIFE ASSURANCE CO. OF CANADA

37
[G.R. No. 23703; September 28, 1925]

Facts:

 January 29, 1910: Sun Life Assurance Co. of Canada issued a 20-year endowment insurance policy on the life of
Hilario Gercio
o insurance company agreed to insure the life of Gercio for the sum of P2,000, to be paid him on February
1, 1930, or if the insured should die before said date, then to his wife, Mrs. Andrea Zialcita, should she
survive him; otherwise to the executors, administrators, or assigns of the insured
o policy did not include any provision reserving to the insured the right to change the beneficiary
 End of 1919: she was convicted of the crime of adultery
 September 4, 1920: a decree of divorce was issued
 March 4, 1922: Gercio formally notified the Sun Life that he had revoked his donation in favor of Andrea Zialcita,
and that he had designated in her stead his present wife, Adela Garcia de Gercio, as the beneficiary of the policy
o Sun Life refused
 Gercio filed a petition for mandamus to compel Sun Life
o Trial Court: favored Gercio

Issue: W/N Gercio has the right to change the beneficiary of the policy

Ruling:

NO. Dismissed.

 The wife has an insurable interest in the life of her husband.


 The beneficiary has an absolute vested interest in the policy from the date of its issuance and delivery. So when a
policy of life insurance is taken out by the husband in which the wife is named as beneficiary, she has a subsisting
interest in the policy
o applies to a policy to which there are attached the incidents of a loan value, cash surrender value, an
automatic extension by premiums paid, and to an endowment policy, as well as to an ordinary life
insurance policy.
 If the husband wishes to retain to himself the control and ownership of the policy he may so provide in the
policy.
o But if the policy contains no provision authorizing a change of beneficiary without the beneficiary’s
consent, the insured cannot make such change.
 Accordingly, it is held that a life insurance policy of a husband made payable to the wife as beneficiary, is the
separate property of the beneficiary and beyond the control of the husband.
 effect produced by the divorce, the Philippine Divorce Law, Act No. 2710, merely provides in section 9 that the
decree of divorce shall dissolve the community property as soon as such decree becomes final
o absence of a statute to the contrary, that if a policy is taken out upon a husband’s life the wife is named
as beneficiary therein, a subsequent divorce does not destroy her rights under the policy

38
 Neither the husband, nor the wife, nor both together had power to destroy the vested interest of the children in
the policy.

Delfin Nario and Alejandra Santos-Nario v. The Philippine American Life Insurance Company

FACTS:

FACTS:
 June 12, 1959: Philippine American Life Insurance Co. issued a life insurance to Mrs. Alejandra Santos-Mario a life
insurance policy under a 20-year endowment plan, with a face value of P5,000 designating her husband Delfin
Nario and their unemancipation son Ernesto Nario, as her irrevocable beneficiaries
 June, 1963: She submitted her loan application to the life insurance co. with signature of her husband in two
capacities:
o irrevocable beneficiaries
o father-guardian of minor irrevocable beneficiary Ernesto
 Insurance Co. denied asking that the legal guardian must be authorized by the court in a competent guardianship
proceeding
 Upon denial, she opted to surrender her insurance policy in exchange of its cash surrender value of P520 but it
was also denied on the same ground
 September 10, 1963: Mrs. Alejandra Santos-Nario and her husband, Delfin Nario, brought suit against the
Philippine American Life Insurance Co

ISSUE:
Whether the insurer may refuse to grant the loan applicati on and the surrender of the policy claimed by a
father-guardian in behalf of his minor son absent a court authority in aguardianship proceeding.

HELD: YES.
T h e p r o p o s e d t r a n s a c ti o n s i n q u e s ti o n ( p o l i c y l o a n a n d s u r r e n d e r o f p o l i c y ) consti tute acts of
dispositi on or alienati on of property rights and not merely of management or administration because they
involve the incurring or termination of contractual obligations.It appearing that the minor beneficiary's vested interest or
right on the policy exceeds twothousand pesos (P2,000.00), the plaintiffs should have, but, had not, filed
a guardianship bondand a formal applicati on or peti ti on for guardianship
, U n d e r S e c ti o n 7 , R u l e 9 3 o f t h e Revised Rules of Court. Hence, plaintiffs cannot possibly exercise the
powers vested on them, aslegal administrators of their child's property, under articles 320 and 326 of the Civil Code.
Ther e s u l t w o u l d b e t h e s a m e e v e n i f w e r e g a r d e d t h e i n t e r e s t o f t h e w a r d t o b e w o r t h l e s s
t h a n P2,000.00. While the father or mother would in such event be exempt from the duty of fi ling
a bond, and securing judicial appointment, still the parent's authority over the estate of the ward asa legal-guardian
would not extend to acts of encumbrance or disposition, as distinguished fromacts of management or administration

SOCIAL SECURITY SYSTEM vs. CANDELARIA D. DAVAC [G.R. No. L-21642. July 30, 1966.]

Facts:

This is an appeal from the resolution of the Social Security Commission declaring respondent Candelaria Davac as the
person entitled to receive the death benefits payable for the death of Petronilo Davac.

39
The late Petronilo Davac, a former employee of Lianga Bay Logging Co. Inc. became a member of the Social Security
System (SSS for short) on September 1, 1957. He designated respondent Candelaria Davac as his beneficiary and
indicated his relationship to her as that of "wife". When he died, each of the respondents (Candelaria Davac and Lourdes
Tuplano) filed their claims for death benefit with the SSS. It appears from their respective claims and the documents
submitted in support thereof, that the deceased contracted two marriages, the first, with claimant Lourdes Tuplano on
August 29, 1946, who bore him a child, Romeo Davac and the second, with claimant Candelaria Davac on January 18,
1949, with whom he had a minor daughter, Elizabeth Davac. Due to their conflicting claims, the processing thereof was
held in abeyance, whereupon the SSS filed this petition praying that respondents be required to interplead and litigate
between themselves their conflicting claims over the death benefits in question.

Issue:

Whether or not the Social Security Commission acted correctly in declaring respondent Candelaria Davac as the person
entitled to receive the death benefits in question.

Held: yes.

The benefit receivable under the Social Security Act is in the nature of a special privilege or an arrangement secured by
the law, pursuant to the policy of the State to provide social security to the workingmen. The amounts that may thus be
received cannot be considered as property earned by the member during

his lifetime, and, hence, do not form part of the properties of the conjugal partnership or of the estate of the said
member. They are disbursed from a public special fund created by Congress pursuant to the declared policy of the
Republic "to develop, establish gradually and perfect a social security system which . . . shall provide protection against
the hazards of disability, sickness, old age and death." (Section 1, Republic Act No. 1792.) Consequently, if there is a
named beneficiary and the designation is not invalid, it is not the heirs of the employee who are entitled to receive the
benefits, unless they are the designated beneficiaries themselves. It is only when there is no designated beneficiary or
when the designation is void that the laws of succession become applicable. The Social Security Act is not a law of
succession.

Vda. De Consuegra v. GSIS - Retirement Insurance Benefits

37 SCRA 315

Facts:

> Jose Consuegra was employed as a shop foreman of the Office of the District Engineer in Surigao Del Norte.

> When he was still alive, he contracted two marriages:

o First – Rosario Diaz; 2 children = Jose Consuegra Jr. and Pedro but both predeceased him

o 2nd – Basilia Berdin; 7 children. (this was contracted in GF while the first marriage subsisted)

> Being a GSIS member when he died, the proceeds of his life insurance were paid by the GSIS to Berdin and her
children who were the beneficiaries named in the policy.
40
> Since he was in the gov’t service for 22.5028 years, he was entitled to retirement insurance benefits, for which no
beneficiary was designated.

> Both families filed their claims with the GSIS, which ruled that the legal heirs were Diaz who is entitled to one-half or
8/16 of the retirement benefits and Berdin and her children were entitled to the remaining half, each to receive an equal
share of 1/16.

> Berdin went to CFI on appeal. CFI affirmed GSIS decision.

Issue:

To whom should the retirement insurance benefits be paid?

Held:

Both families are entitled to half of the retirement benefits.

The beneficiary named in the life insurance does NOT automatically become the beneficiary in the retirement insurance.
When Consuegra, during the early part of 1943, or before 1943, designated his beneficiaries in his life insurance, he
could NOT have intended those beneficiaries of his life insurance as also the beneficiaries of his retirement insurance
because the provisions on retirement insurance under the GSIS came about only when CA 186 was amended by RA 660
on June 18, 1951.

Sec. 11(b) clearly indicates that there is need for the employee to file an application for retirement insurance benefits
when he becomes a GSIS member and to state his beneficiary. The life insurance and the retirement insurance are two
separate and distinct systems of benefits paid out from 2 separate and distinct funds.

In case of failure to name a beneficiary in an insurance policy, the proceeds will accrue to the estate of the insured. And
when there exists two marriages, each family will be entitled to one-half of the estate.

Lopez v. Del Rosario

44 PHIL 98

Facts:

> Benita Del Rosario is the owner of a bonded warehouse in Manila where copra and other merchandise are deposited.

> Among those who had copra deposited in the warehouse was Froilan Lopez, the owner of 14 warehouse receipts with
a declared value of P107,990.40 in his name.

41
> Del Rosario secured insurance on the warehouse and its contents with 5 different insurance companies in the amount
of P404,800.

> All policies were in the name of Del Rosario, except for one (with Nat’l Insurance Co.) for 40T, in favor of Compania
Copra de Tayabas.

> The warehouse and its contents were destroyed by fire. When Bayne, a fire loss adjuster, failed to effect a settlement
between the Insurance companies and Del Rosario, the latter authorized Atty. Fisher to negotiate with the Companies.

> An agreement was reached to submit the matter to arbitration. The claims by different people who had stored copra
in the warehouse were settled with the exception of Friolan Lopez.

> A case was filed in CFI by Lopez. The court awarded him the sum of P88,492.21 with legal interest.

Issue:

Whether or not Del Rosario acted as the agent of Lopez in taking out the insurance on the contents of the warehouse or
whether she acted as the reinsurer of the copra.

Held:

She acted as the agent of Lopez.

The agency can be deduced from the warehouse receipts, the insurance policies and the circumstances surrounding the
transaction. Under any aspect, Del Rosario is liable. The law is that a policy effected by a bailee and covering by its
terms in his own property and property held in trust, inures, in the event of loss, equally and proportionately to the
benefit of all owners of the property insured. Even if one secured insurance covering his own goods and goods stored
with him, and even if the owner of the stored goods did not request or know the insurance, and did not ratify it before
the payment of the loss, it has been held by a reputable court that the warehouseman is liable to the owner of such
stored goods for his share.

In a case of contributing policies, adjustments of loss made by an expert or by a board of arbitrators may be submitted to
the court NOT as evidence of the facts stated therein, or as obligatory, but for the purpose of assisting the court in
calculating the amount of liability.

Cha V. CA (1997)

G.R. No. 124520 August 18, 1997

FACTS:  Spouses Nilo Cha and Stella Uy-Cha and CKS Development Corporation entered a 1 year lease contract with a
stipulation not to insure against fire the chattels, merchandise, textiles, goods and effects placed at any stall or store or
42
space in the leased premises without first obtaining the written consent and approval of the lessor. But it insured against
loss by fire their merchandise inside the leased premises for P500,000 with the United Insurance Co., Inc. without the
written consent of CKS  On the day the lease contract was to expire, fire broke out inside the leased premises and CKS
learning that the spouses procured an insurance wrote to United to have the proceeds be paid directly to them. But
United refused so CKS filed against Spouses Cha and United. 

RTC: United to pay CKS the amount of P335,063.11 and Spouses Cha to pay P50,000 as exemplary damages, P20,000 as
attorney’s fees and costs of suit 

CA: deleted exemplary damages and attorney’s fees

ISSUE: W/N the CKS has insurable interest because the spouses Cha violated the stipulation

HELD: NO. CA set aside. Awarding the proceeds to spouses Cha.  Sec. 18. No contract or policy of insurance on
property shall be enforceable except for the benefit of some person having an insurable interest in the property insured

 A non-life insurance policy such as the fire insurance policy taken by petitioner-spouses over their merchandise is
primarily a contract of indemnity. Insurable interest in the property insured must exist a t the time the insurance takes
effect and at the time the loss occurs. The basis of such requirement of insurable interest in property insured is based on
sound public policy: to prevent a person from taking out an insurance policy on property upon which he has no insurable
interest and collecting the proceeds of said policy in case of loss of the property. In such a case, the contract of insurance
is a mere wager which is void under Section 25 of the Insurance Code.  SECTION 25. Every stipulation in a policy of
Insurance for the payment of loss, whether the person insured has or has not any interest in the property insured, or
that the policy shall be received as proof of such interest, and every policy executed by way of gaming or wagering, is
void  Section 17. The measure of an insurable interest in property is the extent to which the insured might be
damnified by loss of injury thereof  The automatic assignment of the policy to CKS under the provision of the lease
contract previously quoted is void for being contrary to law and/or public policy. The proceeds of the fire insurance
policy thus rightfully belong to the spouses. The liability of the Cha spouses to CKS for violating their lease contract in
that Cha spouses obtained a fire insurance policy over their own merchandise, without the consent of CKS, is a separate
and distinct issue which we do not resolve in this case.

Garcia v. HongKong Fire and Marine Insurance Co.

Facts:

> Garcia had his merchandise insured by Hongkong Fire and Marine Insurance Co.

> The insurance company however made a mistake and issued a policy covering the building where the merchandise
was stored. (The building was not owned by Garcia)

> The policy was written in English, of which Garcia was ignorant, so he could not have noticed the error of the insurance
company.

> Said policy was later on assigned by Garcia to PNB to secure a loan. PNB acknowledged receipt of said policy, referring
to it as a policy covering the merchandise.

43
> The insurance company made the necessary endorsements to PNB.

> The building which housed the merchandise was later razed by fire. The insurance company refused to pay due to the
fact that the policy indicates insurance on the building and not on the merchandise.

Issue:

Whether or not Garcia can collect.

Held:

YES.

The defense of the insurer is purely technical. The mistake was obviously on the part of the insurer when it issued a
wrong policy. It cannot deny such allegation due to the fact that it even confirmed with PNB the nature of said policy
when it was endorsed. Garcia could not have noticed the mistake due to his ignorance of the English language.

K. S. YOUNG vs. THE MIDLAND TEXTILE INSURANCE COMPANY

[G.R. No. 9370; March 31, 1915]

Warranties: violation of material warranty, effect

Facts:

The purpose of the present action is to recover the sum of P3,000 upon an insurance policy. The lower
court rendered a judgment in favor of the plaintiff and against the defendant for the sum
of P2,708.78, and costs. From that judgment the defendant appealed to this court. The undisputed
facts upon which said action is based are as follows: The plaintiff occupied a building at ‘321 Calle
Claveria, as a residence and bodega (storehouse).On the 29th of May, 1912, the defendant, in
consideration of the payment of a premium of P60, entered into a contract of insurance with the
plaintiff promising to pay to the plaintiff the sum of P3,000, in case said residence and bodega and
contents should be destroyed by fire. One of the conditions of said contract was that no hazardous
goods be stored or kept in the building. On the 4th or 5th of February, 1913, the plaintiff placed in said
residence and bodega three boxes which belonged to him and which were filled with fireworks for the
celebration of the Chinese new year. On the 18th day of March, 1913, said residence and bodega and
the contents thereof were partially destroyed. Fireworks were found in a part of the building not
destroyed by the fire; that they in no way contributed to the fire, or to the loss occasioned thereby.

Issue:

44
Whether or not the placing of said fireworks in the building insured, under the conditions above
enumerated, they being “hazardous goods,” is a violation of the terms of the contract of insurance.

Ruling:

Yes. The word “stored” has been defined to be a deposit in a store or warehouse for preservation or
safe keeping; to put away for future use, especially for future consumption; to place in a warehouse or
other place of deposit for safe keeping. Said definition does not include a deposit in a store, in small
quantities, for daily use. “Daily use” precludes the idea of deposit for preservation or safe keeping, as
well as a deposit for future consumption or safe keeping. A violation of the terms of a contract of
insurance, by either party, will constitute the basis for a termination of the contractual relations, at
the election of the other. The right to terminate the contractual relations exists even though the
violation was not the direct cause of the loss. In the present case, the deposit of the “hazardous
goods,” in the building insured, was a violation of the terms of the contract. Although the hazardous
goods did not contribute to the loss, the insurer, at his election, was relieved from liability Said deposit
created a new risk, not included in the terms of the contract. The insurer had neither been paid, nor
had he entered into a contract, to cover the increased risk.

PHIL. HEALTH CARE PROVIDERS, INC vs. COMMISSIONER OF INTERNAL REVENUE

GR. NO. 167330 September 18, 2009, SPECIAL FIRST DIVISION (CORONA, J.)

FACTS:

Petitioner is a domestic corporation whose primary purpose is to establish, maintain, conduct and operate a prepaid
group practice health care delivery system or a health maintenance organization to take care of the sick and disabled
persons enrolled in the health care plan and to provide for the administrative, legal, and financial responsibilities of the
organization. On January 27, 2000, respondent CIR sent petitioner a formal deman letter and the corresponding
assessment notices demanding the payment of deficiency taxes, including surcharges and interest, for the taxable years
1996 and 1997 in the total amount of P224,702,641.18. The deficiency assessment was imposed on petitioner’s health
care agreement with the members of its health care program pursuant to Section 185 of the 1997 Tax Code. Petitioner
protested the assessment in a letter dated February 23, 2000. As respondent did not act on the protest, petitioner filed a
petition for review in the Court of Tax Appeals (CTA) seeking the cancellation of the deficiency VAT and DST assessments.

ISSUES:

Whether or not Philippine Health Care Providers, Inc. engaged in insurance business.

HELD:

NO. Health Maintenance Organizations are not engaged in the insurance business. The SC said in June 12, 2008 decision
that it is irrelevant that petitioner is an HMO and not an insurer because its agreements are treated as insurance
contracts and the DST is not a tax on the business but an excise on the privilege, opportunity or facility used in the
transaction of the business. Petitioner, however, submits that it is of critical importance to characterize the business it is
engaged in, that is, to determine whether it is an HMO or an insurance company, as this distinction is indispensable in
turn to the issue of whether or not it is liable for DST on its health care agreements. Petitioner is admittedly an HMO.
45
Under RA 7878 an HMO is “an entity that provides, offers or arranges for coverage of designated health services needed
by plan members for a fixed prepaid premium. The payments do not vary with the extent, frequency or type of services
provided. Section 2 (2) of PD 1460 enumerates what constitutes “doing an insurance business” or “transacting an
insurance business”which are making or proposing to make, as insurer, any insurance contract; making or proposing to
make, as surety, any contract of suretyship as a vocation and not as merely incidental to any other legitimate business or
activity of the surety; doing any kind of business, including a reinsurance business, specifically recognized as constituting
the doing of an insurance business within the meaning of this Code; doing or proposing to do any business in substance
equivalent to any of the foregoing in a manner designed to evade the provisions of this Code.

Overall, petitioner appears to provide insurance-type benefits to its members (with respect to its curative medical
services), but these are incidental to the principal activity of providing them medical care. The “insurance-like” aspect of
petitioner’s business is miniscule compared to its noninsurance activities. Therefore, since it substantially provides health
care services rather than insurance services, it cannot be considered as being in the insurance business.

PHILAMCARE HEALTH SYSTEMS, INC. vs. COURT OF APPEALS G.R. No. 125678, 18 March 2002

FACTS: Ernani Trinos obtained a health care coverage with petitioner Philamcare. Under the agreement, Trinos was
entitled to avail of hospitalization benefits, whether ordinary or emergency, listed therein. He was entitled to avail of
“out-patient benefits” such as annual physical examinations, preventive health care and other out-patient services.
During the period of coverage, Trinos suffered a heart attack and was hospitalized for one month. During this time, his
wife, Julita Trinos, tried to claim the benefits under the health care agreement but petitioner company denied her claim
on the ground that the Health Care Agreement was void because there was concealment regarding Ernani’s medical
history. Doctors allegedly discovered at the time of Ernani’s confinement that he was hypertensive, diabetic and
asthmatic, contrary to his answer in the application form. Thus Julita paid the hospitalization expenses herself. When
Ernani died, Julita instituted with the RTC of Manila an action for damages against petitioner and its president. She asked
for reimbursement of her expenses plus moral damages and attorney’s fees.

ISSUE: Whether or not the petitioner is liable

HELD: YES. The health care agreement was in the nature of non-life insurance, which is primarily a contract of indemnity.
Once the

member incurs hospital, medical or any other expense arising from sickness, injury or other stipulated contingent, the
health care provider must pay for the same to the extent agreed upon under the contract. Petitioner alleges that
respondent was not the legal wife of the deceased member considering that at the time of their marriage, the deceased
was previously married to another woman who was still alive. The health care agreement is in the nature of a contract of
indemnity. Hence, payment should be made to the party who incurred the expenses. It is not controverted that
respondent paid all the hospital and medical expenses. She is therefore entitled to reimbursement. The records
adequately prove the expenses incurred by respondent for the deceased’s hospitalization, medication and the
professional fees of the attending physicians. WHEREFORE, in view of the foregoing, the petition is DENIED. The assailed
decision of the Court of Appeals dated December 14, 1995 is AFFIRMED.

Blue Cross v Olivares G.R. No. 169737, February 12, 2008

J. Corona

46
Facts:

Neomi Olivares applied for a health care program with Blue Cross for the amount of 12,000 pesos. 38 days after she
applied, she suffered from a stroke. Ailments due to “pre-existing conditions” were excluded from the coverage. She was
confined in Medical City and discharged with a bill of Php 34,000. Blue Cross refused to pay unless she had her
physician’s certification that she was suffering from a pre-existing condition. When Blue Cross still refused to pay, she
filed suit in the MTC. The health care company rebutted by saying that the physician didn’t disclose the condition due to
the patient’s invocation of the doctor-client privilege. The MTC dismissed for a lack of cause of action because the
physician didn’t disclose the condition. In the RTC, the spouses were awarded the amount of the hospital bills plus
60,000 in damages. This was under the ratio that the burden to prove that Neomi had a pre-existing condition was under
Blue Cross. The CA denied the motion for reconsideration of the health care company.

Issues:

1. Whether petitioner was able to prove that respondent Neomi's stroke was caused by a pre-existing condition and
therefore was excluded from the coverage of the health care agreement.

Held: No.

Ratio:

a health care agreement is in the nature of a non-life insurance. It is an established rule in insurance contracts that when
their terms contain limitations on liability, they should be construed strictly against the insurer. These are contracts of
adhesion the terms of which must be interpreted and enforced stringently against the insurer which prepared the
contract. This doctrine is equally applicable to health care agreements.”

Petitioner never presented evidence to prove its presumption that the Doctor’s report would work against Neomi. They
only perceived that the invocation of the privilege made the report adverse to Neomi and such was a disreputable
presumption. They should have made an independent assessment of Neomi’s condition when it failed to obtain the
report. They shouldn’t have waited for the attending physician’s report to come out.

San Miguel Brewery, etc. v. Law Union and Rock Insurance Co., (Ltd.), et.al.
G.R. No. L-14300 January 19, 1920
FACTS
(1916) A property originally owned by D.P. Dunn was mortgaged to plaintiff to secure a debt of P10,000. Dunn agreed to
keep the property insured at his expense to insurance companies to be selected by plaintiff; to receive the proceeds of
the insurance in case of loss and to retain such part necessary to cover the mortgage debt. Brias, general manager of
plaintiff made a verbal application to Law Union for P15,000. In the application, Brias stated that the company is
interested only as a mortgagee.

No information was asked as to who was the owner of the property, and no information upon this point was given. Law
Union assumes one-half of the risk only and the other half was issued by co-
defendant insurance company “Filipinas” Compania de Seguros.
(1917) the policies were renewed, renewal premiums being paid by plaintiff, supposedly for the account of the owner.
Thereafter, Dunn sold the mortgage property to co-defendant Henry Harding, but not assignment of the insurance, or of
the insurance policies, was at any time made to him. The property was destroyed by fire and plaintiff wanted to recover
from the insurance
47
the proceeds to the extent of its mortgage credit with the balance to go to Harding. Defendant insurance companies

contended that they were not liable to Harding because their liability under the policies was limited to the insurable
interests of plaintiff only. Plaintiff eventually reached a settlement with the insurance companies and was paid the
balance of its mortgage credit. Harding was left to fend for himself.
Ruling of the Lower Court
Concluded that Harding had no right of action whatever against the insurance companies.
ISSUE
Are the insurance companies liable to defendant Harding?
RULING
No. Under the Insurance Act, the measure of insurable interest in the property is the extent to which the insured might
be damnified by the loss or injury thereof. Also it is provided in that

the insurance shall be applied exclusively to the proper interest of the person in whose name it is made.

Undoubtedly, SMB as the mortgagee of the property, had an insurable interest therein; but it could not, in any event,
recover upon the two policies an amount in excess of its mortgage credit. An additional insuperable obstacle is that the
ownership over the property had changed prior to the loss, without corresponding change having been effected in the
policy of insurance. In section 19 of the Insurance Act we find it stated that "a change of interest in any part of a thing
insured unaccompanied by a corresponding change of interest in the insurance, suspends the insurance to an equivalent
extent, until the interest in the thing and the interest in the insurance are vested in the same person." Again in section 55
it is declared that "the mere transfer of a thing insured does not transfer the policy, but suspends it until the same
person becomes the owner of both the policy and the thing insured."

MA. LOURDES S. FLORENDO,


Petitioner,vs.
PHILAM PLANS, INC., PERLA ABCEDE MA. CELESTE ABCEDE,
Respondents.
FACTS:
Manuel Florendo filed an application for comprehensive pension plan with respondent PhilamPlans, Inc. (Philam Plans) Manuel signed the
application and leftto Perla the task of supplying
the information needed in the application. Respondent Ma. Celeste Abcede, Perla’s daughter,
signed the application as sales counselor. Philam Plans issued Pension Plan Agreement toManuel, with petitioner Ma. Lourdes S. Florendo, his
wife, as beneficiary. In time, Manuel paidhis quarterly premiums. Eleven months later, Manuel died of blood poisoning. Subsequently,Lourdes
filed a claim with Philam Plans for th
e payment of the benefits under her husband’s
plan but Philam Plans declined her claim prompting her to file the present action against thepension plan company before the Regional Trial
Court (RTC) of Quezon City and ruled in favor of Ma. Lourdes. However, the Court of Appeals then reversed the RTC decision. Hence thisappeal.

ISSUE:
Whether or not Ma. Lourdes could claim benefits as the beneficiary of her husband under theinsurance plan despite consideration that her
husband Manuel concealed the true condition of his health.

RULING:
The Supreme Court answers this to the negative and the AFFIRMED in its entirety the decisionof the Court of Appeals.The comprehensive
pension plan that Philam Plans issued contains a one-year incontestabilityperiod. It states:
VIII. INCONTESTABILITY
After this Agreement has remained in force for one (1) year, we can no longer contest for health reasons any claim for
insurance under this Agreement, except for the reason thatinstallment has not been paid (lapsed), or that you are not insurable at the time you

48
bought thispension program by reason of age. If this Agreement lapses but is reinstated afterwards, theone (1) year contestability period shall
start again on the date of approval of your request for reinstatement.The above
incontestability clause
precludes the insurer from disowning liability under thepolicy it issued on the ground of concealment or misrepresentation regarding the health
of theinsured after a year of its issuance.Since
Manuel died on the eleventh month
following the issuance of his plan, the one year incontestability period has not yet set in. Consequently, Philam Plans was not barred from
questioning Lourdes’ entitlement to the benefits of her husband’s pension plan.

QUA CHEE GAN vs. LAW UNION AND ROCK INSURANCE CO., LTD. G.R. No. L-4611, 17 December 1955 FACTS: Plaintiff-
appellee owned four bodegas used for the storage of copra and hemp. These buildings, together with their contents,
were insured with the defendant company since 1937 and the loss made payable to the Philippine National Bank as
mortgage of the hemp and crops, to the extent of its interest.

Sometime after, three of the bodegas, together with the merchandise inside, were completely destroyed by fire of an
undetermined origin. Consequently, Qua Chee Gan notified the insurance company of his loss. The latter conducted an
extensive investigation. The damage was determined to be equivalent to P398,562.81 which was later reduced to the full
amount of the insurance, Php370,000.00. However, the insurance company refused payment, claiming violation of
warranties and conditions, filing of fraudulent claims, and that the fire had been deliberately caused by the insured or by
other persons in connivance with him. If moreover argued that since the bodegas insured had an external wall perimeter
of 500 meters or 1,640 feet, the appellee should have 11 fire hydrants in the compound, and that he actually had only 2
with a further pair nearby, belonging to the municipality of Tabaco.

ISSUE: Whether or not the insurer company is liable

HELD: YES. The SC is in agreement with the trial court that the appellant is barred by waiver (or rather estoppel) to claim
violation of the so-called fire hydrants warranty, for the reason that knowing fully all that the number of hydrants
demanded therein never existed from the very beginning, the appellant nevertheless issued the policies in question
subject to such warranty, and received the corresponding premiums. It would be perilously close to conniving at fraud
upon the insured to allow appellant to claims now as void ab initio the policies that it had issued to the plaintiff without
warining of their fatal defect, of which it was informed, and after it had misled the defendant into believing that the
policies were effective. The contract of insurance is one of perfect good faith (uferrimal fidei) not for the insured alone,
but equally so for the insurer, in fact, it is mere so for the latter, since its dominant bargaining position carries with it
stricter responsibility. We find no reversible error in the judgment appealed from, wherefore the same is hereby
affirmed. Costs against the appellant.

FIELDMEN’S INSURANCE CO. VS. VDA DE SONGCO G.R. NO. 24833 SEPTEMBER 23, 1986

FACTS: FEDERICO SONGCO, OWNER OF A PRIVATE JEEPNEY, WAS INDUCED BY THE AGENT OF FIELDMEN’S INSURANCE
CO. TO APPLY FOR A COMMON CARRIER’S INSURANCE POLICY WHICH WAS LATER RENEWED. DURING THE EFFECTIVITY
OF THE RENEWED POLICY, THE JEEPNEY COLLIDED WITH A CAR RESULTING DEATH TO FEDERICO AND HIS SON. NOW, THE
SURVIVING SPOUSE OF FEDERICO SEEKS RELIEF TO FIELDMEN’S. THE COMPANY REFUSED TO PAY.

ARGUMENTS: THE AGENT OF FIELDMEN’S CONTENDS THAT SINCE THEY ARE NOT GOVERNMENT-OWNED, THEY COULD
DO WHAT THEY PLEASE WHENEVER THEY BELIEVE A VEHICLE IS INSURABLE. SONGCO, ON THE OTHER HAND, ASSERTS
THAT THEY SHOULD BE ENTITLED FOR THE PAYMENT.

ISSUE: WHETHER OR NOT FIELDMEN’S SHOULD BE HELD LIABLE.

49
RULING OF THE COURT: YES. IT IS A WELL-SETTLED JURISPRUDENCE THAT WHERE INEQUITABLE CONDUCT IS SHOWN BY
AN INSURANCE FIRM, IT IS "ESTOPPED FROM ENFORCING FORFEITURES IN ITS FAVOR, IN ORDER TO FORESTALL FRAUD
OR IMPOSITION ON THE INSURED." AS ESTOPPEL IS PRIMARILY BASED ON THE DOCTRINE OF GOOD FAITH AND THE
AVOIDANCE OF HARM THAT WILL BEFALL THE INNOCENT PARTY DUE TO ITS INJURIOUS RELIANCE, THE FAILURE TO APPLY
IT IN THIS CASE WOULD RESULT IN A GROSS TRAVESTY OF JUSTICE. AS STATED IN QUA CHEE GAN CASE, THE CONTRACT
OF INSURANCE IS ONE OF PERFECT GOOD FAITH (UBERIMA FIDES) NOT FOR THE INSURED ALONE, BUT EQUALLY SO FOR
THE INSURER; IN FACT, IT IS MORE SO FOR THE LATTER, SINCE ITS DOMINANT BARGAINING POSITION CARRIES WITH IT
STRICTER RESPONSIBILITY. THIS IS MERELY TO STRESS THAT WHILE THE MORALITY OF THE BUSINESS WORLD IS NOT THE
MORALITY OF INSTITUTIONS OF RECTITUDE LIKE THE PULPIT AND THE ACADEME, IT CANNOT DESCEND SO LOW AS TO
BE ANOTHER NAME FOR GUILE OR DECEPTION. MOREOVER, SHOULD IT HAPPEN THUS, NO COURT OF JUSTICE SHOULD
ALLOW ITSELF TO LEND ITS APPROVAL AND SUPPORT.

IGNACIO SATURNINO vs. THE PHILIPPINE AMERICAN LIFE INSURANCE COMPANY G.R. No. L-16163, 28 February 1963

FACTS: Estefania Saturnino obtained a 20-year endowment non-medical insurance. This kind of policy dispenses with the
medical examination of the applicant usually required in ordinary life policies. However, two months prior to the issuance
of the policy, Saturnino was operated on for cancer, involving mastectomy of the right breast. She did not make a
disclosure thereof in her application for insurance. On the contrary, she stated therein that she did not have, nor had she
ever had, among other ailments listed in the application, cancer or other tumors. Sometime after, Saturnino died of
pneumonia, secondary to influenza. Appellants here, who are her surviving husband and minor child, respectively,
demanded payment of the face value of the policy. The claim was rejected and hence an action was subsequently
instituted.

ISSUE: Whether or not the insured made such false representations of material facts as to avoid the policy

HELD: YES. The Insurance Law provides that “materiality is to be determined not by the event, but solely by the probable
and reasonable influence of the facts upon the party to whom the communication is due, in forming his estimate of the
proposed contract, or in making his inquiries.” The waiver of medical examination renders even more material the
information required of the applicant concerning previous condition of health and diseases suffered, for such
information necessarily constitutes an important factor which the insurer takes into consideration in deciding whether to
issue the policy or not. It is logical to assume that if appellee had been properly apprised of the insured’s medical history
she would at least have been made to undergo medical examination in order to determine her insurability. A
concealment, whether intentional or unintentional, entitles the insurer to rescind the contract of insurance, concealment
being defined as “negligence to communicate that which a

party knows and ought to communicate”. The basis of the rule vitiating the contract in cases of concealment is that it
misleads or deceives the insurer into accepting the risk, or accepting it at the rate of premium agreed upon. The insurer,
relying upon the belief that the assured will disclose every material facts within his actual or presumed knowledge, is
misled into a belief that the circumstance withheld does not exist, and he is thereby induced to estimate the risk upon a
false basis that it does not exist. The judgment appealed from, dismissing the complaint and awarding the return to
appellants of the premium already, paid, with interest at 6% up to January 29, 1959, affirmed, with costs against
appellants.

Great Pacific Life vs. CA gr 11389

50
FACTS:

Great Pacific Life Assurance Corporation (Grepalife) executed a contract of group life insurance with Development Bank
of the Philippines (DBP) wherein Grepalife agreed to insure the lives of eligible housing loan mortgagors of DBP.

One such loan mortgagor is Dr. Wilfredo Leuterio. In an application form, Dr. Leuterio answered questions concerning his
test, attesting among others that he does not have any heart conditions and that he is in good health to the best of his
knowledge.

However, after about a year, Dr. Leuterio died due to “massive cerebral hemorrhage.” When DBP submitted a death claim
to Grepalife, the latter denied the claim, alleging that Dr. Leuterio did not disclose he had been suffering from
hypertension, which caused his death. Allegedly, such non-disclosure constituted concealment that justified the denial of
the claim.

Hence, the widow of the late Dr. Leuterio filed a complaint against Grepalife for “Specific Performance with Damages.”
Both the trial court and the Court of Appeals found in favor of the widow and ordered Grepalife to pay DBP.

ISSUE:

 Whether the CA erred in holding Grepalife liable to DBP as beneficiary in a group life insurance contract from a
complaint filed by the widow of the decedent/mortgagor

HELD:

The rationale of a group of insurance policy of mortgagors, otherwise known as the “mortgage redemption insurance,” is
a device for the protection of both the mortgagee and the mortgagor. On the part of the mortgagee, it has to enter into
such form of contract so that in the event of the unexpected demise of the mortgagor during the subsistence of the
mortgage contract, the proceeds from such insurance will be applied to the payment of the mortgage debt, thereby
relieving the heirs of the mortgagor from paying the obligation. In a similar vein, ample protection is given to the
mortgagor under such a concept so that in the event of death, the mortgage obligation will be extinguished by the
application of the insurance proceeds to the mortgage indebtedness. In this type of policy insurance, the mortgagee is
simply an appointee of the insurance fund. Such loss-payable clause does not make the mortgagee a party to the
contract.

The insured, being the person with whom the contract was made, is primarily the proper person to bring suit thereon.
Subject to some exceptions, insured may thus sue, although the policy is taken wholly or in part for the benefit of
another person, such as a mortgagee.

And since a policy of insurance upon life or health may pass by transfer, will or succession to any person, whether he has
an insurable interest or not, and such person may recover it whatever the insured might have recovered, the widow of
the decedent Dr. Leuterio may file the suit against the insurer, Grepalife.

Argente v. West Coast Life Insurance Co.- Misrepresentation

51 PHIL 725

51
Facts:

> A joint life insurance policy was issued to Bernardo Argente and his wife Vicenta upon payment of premium, by West
Coast.

> On Nov. 18, 1925, during the effectivity of the policy, Vicenta died of cerebral apoplexy. Thereafter, Bernardo claimed
payment but was refused.

> It is admitted that in the Medical Examiner’s report, Vicenta, in response to the question asked by the medical
examiner, her replies were as follows:

o “How frequently do you use beer, wine, spirits and other intoxicants?” she answered “beer only in small quantities”.

o “What physician have you consulted or been treated by within the last 5 years and for what illness or ailment?” she
answered “none”

> It is however, not disputed that in 1924, Vicenta was taken to a hospital for what was first diagnosed as alcoholism and
later changed to manic-depressive psychosis and then again changed to pscyhonuerosis.

Issue:

Whether or not on the basis of the misrepresentations of Vicenta, Bernardo is barred from recovery.

Held:

YES.

The court found that the representations made by Vicenta in his application for life insurance were false with respect to
her state of health and that she knew and was aware that the representations so made by her were false. In an action on
a life insurance policy where the evidence conclusively shows that the answers to questions concerning diseases were
untrue, the truth or falsity of the answer becomes the determining factor.

If the policy was procured by fraudulent misrepresentations, the contract of insurance apparently set forth therein was
never legally existent. It can be fairly assumed that had the true facts been disclosed by the insured, the insurance
would never have been granted.

Yu Pang Cheng v. CA-

52
Facts:

> Yu Pang Eng obtained a life insurance policy naming his brother Yu Pang Cheng as beneficiary.

> Eng subsequently died of medullary carcinoma, Grade 4, advanced and lesser curvature.

> Cheng claims the proceeds of the policy.

> Insurance co. refused payment on the ground that the policy was void due to the concealment.

Issue:

Whether or not the policy is void.

Held:

YES.

In the application for the policy, Eng was asked whether he had been ill or had consulted a doctor due to symptoms or
illnesses enumerated in the questionnaire. He answered “ No”, when in fact he was hospitalized seven months prior to
his application for the said policy.

SUNLIFE ASSURANCE COMPANY OF CANADA vs. COURT OF APPEALS G.R. No. 105135, 22 June 1995

FACTS: Robert John Bacani procured a life insurance contract for himself from petitioner-company, designating his
mother Bernarda Bacani, herein private respondent, as the beneficiary. He was issued a policy valued at P100,000.00
with double indemnity in case of accidental death. Sometime after, the insured died in a plane crash. Bernarda filed a
claim with petitioner, seeking the benefits of the insurance policy taken by her son. However, said insurance company
rejected the claim on the ground that the insured did not disclose material facts relevant to the issuance of the policy,
thus rendering the contract of insurance voidable. Petitioner discovered that two weeks prior to his application for
insurance, the insured was examined and confined at the Lung Center of the Philippines, where he was diagnosed for
renal failure. The RTC, as affirmed by the CA, this fact was concealed, as alleged by the petitioner. But the fact that was
concealed was not the cause of death of the insured and that matters relating to the medical history of the insured is
deemed to be irrelevant since petitioner waived the medical examination prior to the approval and issuance of the
insurance policy.

ISSUE: Whether or not the concealment of such material fact, despite it not being the cause of death of the insured, is
sufficient to render the insurance contract voidable

HELD: YES. Section 26 of the Insurance Code is explicit in requiring a party to a contract of insurance to communicate to
the other, in good faith, all facts within his knowledge which are material to the contract and as to which he makes no
warranty, and which the other has no means of ascertaining. Anent the finding that the facts concealed had no bearing

53
to the cause of death of the insured, it is well settled that the insured need not die of the disease he had failed to
disclose to the insurer. It is sufficient that his non-disclosure misled the insurer in forming his estimates of the risks of the
proposed insurance policy or in making inquiries.

The SC, therefore, ruled that petitioner properly exercised its right to rescind the contract of insurance by reason of the
concealment employed by the insured. It must be emphasized that rescission was exercised within the two-year
contestability period as recognized in Section 48 of The Insurance Code. WHEREFORE, the petition is GRANTED and the
Decision of the Court of Appeals is REVERSED and SET ASIDE.

Argente v. West Coast Life Insurance Co.- Misrepresentation

51 PHIL 725

Facts:

> A joint life insurance policy was issued to Bernardo Argente and his wife Vicenta upon payment of premium, by West
Coast.

> On Nov. 18, 1925, during the effectivity of the policy, Vicenta died of cerebral apoplexy. Thereafter, Bernardo claimed
payment but was refused.

> It is admitted that in the Medical Examiner’s report, Vicenta, in response to the question asked by the medical
examiner, her replies were as follows:

o “How frequently do you use beer, wine, spirits and other intoxicants?” she answered “beer only in small quantities”.

o “What physician have you consulted or been treated by within the last 5 years and for what illness or ailment?” she
answered “none”

> It is however, not disputed that in 1924, Vicenta was taken to a hospital for what was first diagnosed as alcoholism and
later changed to manic-depressive psychosis and then again changed to pscyhonuerosis.

Issue:

Whether or not on the basis of the misrepresentations of Vicenta, Bernardo is barred from recovery.

Held:

YES.

The court found that the representations made by Vicenta in his application for life insurance were false with respect to
her state of health and that she knew and was aware that the representations so made by her were false. In an action on
a life insurance policy where the evidence conclusively shows that the answers to questions concerning diseases were
untrue, the truth or falsity of the answer becomes the determining factor.
54
If the policy was procured by fraudulent misrepresentations, the contract of insurance apparently set forth therein was
never legally existent. It can be fairly assumed that had the true facts been disclosed by the insured, the insurance
would never have been granted.

THELMA VDA. DE CANILANG vs. COURT OF APPEALS G.R. No. 92492, 17 June 1993

FACTS: Jaime Canilang applied for a “non-medical” insurance policy with respondent Great Pacific Life Assurance
Company naming his wife, Thelma Canilang as his beneficiary. But he did not disclose the fact that he was diagnosed as
suffering from sinus tachycardia and that he has consulted a doctor twice. Jaime was issued an ordinary life insurance
policy with the face value of P19,700.00. Jaime died of “congestive heart failure”, “anemia”, and “chronic anemia”.
Petitioner widow and beneficiary of the insured, filed a claim with Great Pacific which the insurer denied upon the
ground that the insured had concealed material information from it. Hence, Thelma filed a complaint against Great
Pacific with the Insurance Commission for recovery of the insurance proceeds.

ISSUE: Whether or not the non-disclosure of certain facts about the insured’s previous health conditions is material to
warrant the denial of the claims of Thelma Canilang

HELD: YES. The SC agreed with the Court of Appeals that the information which Jaime Canilang failed to disclose was
material to the ability of Great Pacific to estimate the probable risk he

presented as a subject of life insurance. Had Canilang disclosed his visits to his doctor, the diagnosis made and medicines
prescribed by such doctor, in the insurance application, it may be reasonably assumed that Great Pacific would have
made further inquiries and would have probably refused to issue a non-medical insurance policy or, at the very least,
required a higher premium for the same coverage. The materiality of the information withheld by Great Pacific did not
depend upon the state of mind of Jaime Canilang. A man’s state of mind or subjective belief is not capable of proof in our
judicial process, except through proof of external acts or failure to act from which inferences as to his subjective belief
may be reasonably drawn. Neither does materiality depend upon the actual or physical events which ensure. Materiality
relates rather to the “probable and reasonable influence of the facts” upon the party to whom the communication
should have been made, in assessing the risk involved in making or omitting to make further inquiries and in accepting
the application for insurance; that “probable and reasonable influence of the facts” concealed must, of course, be
determined objectively, by the judge ultimately. WHEREFORE, the Petition for Review is DENIED for lack of merit and the
Decision of the Court of Appeals dated 16 October 1989 in C.A.-G.R. SP No. 08696 is hereby AFFIRMED. No
pronouncement as to the costs.

Ng Gan Zee vs Asian Crusader Life Insurance

GR No. L-30685, may 30, 1983

Facts:

On May 12, 1962, Kwong Nam applied for a 20-year endowment insurance on his life for the sum of P20,000.00, with his
wife, appellee Ng Gan Zee, as beneficiary. On the same date, appellant, upon receipt of the required premium from the
insured, approved the application and issued the corresponding policy.

55
Upon Kwong Nam’s death due to cancer of the liver with metastasis, appellant denied the claim on the ground that the
answers given by the insured to the questions appearing in his application for life insurance were untrue. Appellant
further maintains that when the insured was examined in connection with his application for life insurance, he gave the
appellant’s medical examiner false and misleading information as to his aliment and previous operation. Appellant
argues that the insured’s statement in his application that a tumor” hard and of a hen’s egg size” was removed during
said operation, constituted material concealment.

Issue:

Whether the insurance company, because of the insured’s representation, was mislead or deceived into entering the
contract.

Ruling:

No. It bears emphasis that Kwong Nam had informed the appellant’s medical examiner that the tumor for which he was
operated on was “associated with ulcer of the stomach” and in the absence of evidence that the insured had sufficient
medical knowledge as to enable him to distinguish between peptic ulcer and a tumor, his statement should be construed
as an expression made in good faith of his belief as to the nature of his ailment and operation. Thus, “concealment exists
where the assured had knowledge of a fact material to the risk, and honesty, good faith and fair dealing requires that he
should communicate it to the assurer, but he designedly and intentionally withholds the same.” Indeed kwong Nam’s
statement must be presumed to have been made by him without knowledge of its incorrectness and without any
deliberate intent on his part to mislead the appellant.

Also, it has been held that where, upon the face of the application, a question appears to be not answered at all or to be
imperfectly answered, and the insurer issued a policy

without any further inquiry, they waive the imperfection of the answer and render the omission to answer more fully
immaterial.

Tan Chay Heng vs. The West Coast Life Insurance


Facts:
Tan Chay Heng, as beneficiary of Tan Ceang, filed an action for insurance claim against West
Coast on January 4, 1926. On February 27, 1926, defendant made its original Answer
consisting of general and specific denial and was amended on August 31, 1926. In such general
and specific denial, defendant raises the defense of vitiation of consent through fraud; therefore,
it alleges that there was no contract between Tan Ceang and itself to begin with. Tan Chay
Heng filed a demurrer to the special defense of defendant contending that according to Section
47 of the Insurance Act, “whenever a right to rescind a contract of insurance is given to the
insurer, such right must be exercised previous to the commencement of an action on the
contract.” The trial court granted the demurrer and rendered a decision in favor of Tan Chay
Heng.

Issue :
Whether or not Section 47 of the Insurance Act applies in the case at bar

Ruling:
No. In the instant case, it will be noted that even in its prayer, the defendant does not seek to
have the alleged insurance contract rescinded. It denies that it ever made any contract of
insurance on the life of Tan Ceang or that any such a contract ever existed, and that is the
question which it seeks to have litigated by its special defense. In the very nature of things, if the
defendant never made or entered into the contract in question, there is no contract to rescind,
56
and, hence, section 47 upon which the lower court based its decision in sustaining the demurrer
does not apply.

EMILIO TAN vs. COURT OF APPEALS G.R. No. 48049, 29 June 1989

FACTS: Tan Lee Siong, father of herein petitioners, applied for life insurance in the amount of P80,000.00 with
respondent company Philippine American Life Insurance Company. Said application was approved and a corresponding
policy was issued effective November 5, 1973, with petitioners as the beneficiaries. On April 26, 1975, Tan Lee Siong died
of hepatoma. Hence, petitioners filed with respondent company their claim for the proceeds of the life insurance policy.
However, the insurance company denied the said claim and rescinded the policy by reason of the alleged
misrepresentation and concealment of material facts made by the deceased Tan Lee Siong in his application for
insurance. The premiums paid on the policy were thereupon refunded. The petitioners contend that the respondent
company no longer had the right to rescind the contract of insurance as rescission must allegedly be done during the
lifetime of the insured within two years and prior to the commencement of action.

ISSUE: Whether or not the insurance company has the right to rescind the contract of insurance despite the presence of
an incontestability clause

HELD: YES. The so-called “incontestability clause” precludes the insurer from raising the defenses of false representations
or concealment of material facts insofar as health and previous diseases are concerned if the insurance has been in force
for at least two years during the insured’s lifetime. The phrase “during the lifetime” found in Section 48 of the Insurance
Law simply means that the policy is no longer considered in force after the insured has died. The key phrase in the
second paragraph of Section 48 is “for a period of two years”. The policy was issued on November 6, 1973 and the
insured died on April 26, 1975. The policy was thus in force for a period of only one year and five months. Considering
that the insured died before the two-year period has lapsed, respondent company is not, therefore, barred from proving
that the policy is ab initio by reason of the insured’s fraudulent concealment or misrepresentation. Moreover,
respondent company rescinded the contract of insurance and refunded the premiums paid on November 11, 1975,
previous to the commencement of this action on November 27, 1975. WHEREFORE, the petition is hereby DENIED for
lack of merit. The questioned decision of the Court of Appeals is AFFIRMED.

PACIFIC BANKING CORPORATION vs. COURT OF APPEALS and ORIENTAL ASSURANCE CORPORATION

G.R. No. L-41014 November 28, 1988PARAS, J.:

Case # 58

FACTS: Fire Policy, an open policy, was issued to the Paramount Shirt Manufacturing Co. by which private respondent
Oriental Assurance Corporation bound itself to indemnify the insured for any loss or damage caused by fire to its
property.

Said policy was duly endorsed to petitioner as mortgagee/ trustor of the properties insured, with the knowledge and
consent of private respondent to the effect that "loss if any under the policy is payable to the Pacific Banking
Corporation".

While the aforesaid policy was in full force and effect, a fire broke out on the subject premises destroying the goods .
Petitioner sent a letter of demand to private respondent for indemnity .

57
At the trial, petitioner presented evidence, a communication of the insurance adjuster to Asian Surety Insurance Co., Inc.,
revealing undeclared co-insurances with the following: with Wellington Insurance; with Empire Surety and with Asian
Surety; undertaken by insured Paramount on the same property covered by its policy with private

respondent whereas the only co-insurances declared in the subject policy are those of with Malayan, with South Sea and
with Victory.

ARGUMENTS:

RESPONDENT: Private respondent raised the defense of fraud and/or violation of Condition No. 3 in the Policy, in the
form of nondeclaration of co-insurances.

PETITIONER: The allegation of fraud is but a mere inference or suspicion.

ISSUE: Whether the plaintiff if guilty of fraud?

RULING: It is not disputed that the insured failed to reveal before the loss three other insurances. By reason of said
unrevealed insurances, the insured had been guilty of a false declaration; a clear misrepresentation and a vital one
because where the insured had been asked to reveal but did not, that was deception. Otherwise stated, had the insurer
known that there were many co-insurances, it could have hesitated or plainly desisted from entering into such contract.
Hence, the insured was guilty of clear fraud

DISCUSSION: Concrete evidence of fraud or false declaration by the insured was furnished by the petitioner itself when
the facts alleged in the policy under clauses "Co-Insurances Declared" and "Other Insurance Clause" are materially
different from the actual number of coinsurances taken over the subject property. Consequently, "the whole foundation
of the contract fails, the risk does not attach and the policy never becomes a contract between the parties.
Representations of facts are the foundation of the contract and if the foundation does not exist, the superstructure does
not arise. Falsehood in such representations is not shown to vary or add to the contract, or to terminate a contract which
has once been made, but to show that no contract has ever existed.

Fortune Insurance and Surety Co., Inc., vs. CA [G.R. No. 115278, May 23, 1995]

Facts: On June 29, 1987, Producer’s Bank of the Philippines’armored vehicle was robbed, in transit, of seven hundred
twenty-five thousand pesos (Php 725,000.00) that it was transferring from its branch in Pasay to its main branch in
Makati. To mitigate their loss, they claim the amount from their insurer, namely Fortune Insurance and Surety Co..

Fortune Insurance, however, assails that the general exemptionclause in the Casualty Insurance coverage had a general
exemptionclause, to wit:

GENERAL EXCEPTIONS

The company shall not be liable under this policy in respect of

xxx xxx xxx

(b) any loss caused by any dishonest, fraudulent or criminal act of the insured or any officer, employee, partner, director,
trustee or authorized representative of the Insured whether acting alone or in conjunction with others. . . .

And, since the driver (Magalong) and security guard (Atiga) of the armored vehicle were charged with three others as
liable for the robbery, Fortune denies Producer’s Bank of its insurance claim.
58
The trial court and the court appeals ruled in favor of recovery, hence, the case at bar.

Issue: Whether recovery is precluded under the general exemption clause.

Held: Yes, recovery is precluded under the general exemption clause. Howsoever viewed, Producers entrusted the three
with the specific duty to safely transfer the money to its head office, with Alampay to be responsible for its custody in
transit; Magalong to drive the armored vehicle which would carry the money; and Atiga to provide the needed security
for the money, the vehicle, and his two other companions. In short, for these particular tasks, the three acted as agents
of Producers. A "representative" is defined as one who represents or stands in the place of another; one who represents
others or another

in a special capacity, as an agent, and is interchangeable with "agent." 23

In view of the foregoing, Fortune is exempt from liability under the general exceptions clause of the insurance policy.

GEAGONIA vs. CA

[G.R. No. 114427 February 6, 1995]

Facts:

Geagonia, owner of a store, obtained from Country Bankers fire insurance policy for P100,000.00. The 1 year policy and
covered the stock trading of dry goods.

The policy noted the requirement that

“3. The insured shall give notice to the Company of any insurance or insurances already effected, or which may
subsequently be effected, covering any of the property or properties consisting of stocks in trade, goods in process
and/or inventories only hereby insured, and unless notice be given and the particulars of such insurance or insurances be
stated therein or endorsed in this policy pursuant to Section 50 of the Insurance Code, by or on behalf of the Company
before the occurrence of any loss or damage, all benefits under this policy shall be deemed forfeited, provided however,
that this condition shall not apply when the total insurance or insurances in force at the time of the loss or damage is not
more than P200,000.00.”

The petitioners’ stocks were destroyed by fire. He then filed a claim which was subsequently denied because the
petitioner’s stocks were covered by two other fire insurance policies for Php 200,000 issued by PFIC. The basis of the
private respondent’s denial was the petitioner’s alleged violation of Condition 3 of the policy.

Geagonia then filed a complaint against the private respondent in the Insurance Commission for the recovery of
P100,000.00 under fire insurance policy and damages. He claimed that he knew the existence of the other two policies.
But, he said that he had no knowledge of the provision in the private respondent’s policy requiring him to inform it of the
prior policies and this requirement was not mentioned to him by the private respondent’s agent.

Issues:

3. WON the petitioner had not disclosed the two insurance policies when he obtained the fire insurance and
thereby violated Condition 3 of the policy.
4. WON he is prohibited from recovering

59
Held:

3. Yes.
4. No.

Petition Granted

Ratio:

3. The court agreed with the CA that the petitioner knew of the prior policies issued by the PFIC. His letter of 18
January 1991 to the private respondent conclusively proves this knowledge. His testimony to the contrary before
the Insurance Commissioner and which the latter relied upon cannot prevail over a written admission made ante
litem motam. It was, indeed, incredible that he did not know about the prior policies since these policies were
not new or original.
4. Stated differently, provisions, conditions or exceptions in policies which tend to work a forfeiture of insurance
policies should be construed most strictly against those for whose benefits they are inserted, and most favorably
toward those against whom they are intended to operate.

With these principles in mind, Condition 3 of the subject policy is not totally free from ambiguity and must be
meticulously analyzed. Such analysis leads us to conclude that (a) the prohibition applies only to double insurance, and
(b) the nullity of the policy shall only be to the extent exceeding P200,000.00 of the total policies obtained.

Furthermore, by stating within Condition 3 itself that such condition shall not apply if the total insurance in force at the
time of loss does not exceed P200,000.00, the private respondent was amenable to assume a co-insurer’s liability up to a
loss not exceeding P200,000.00. What it had in mind was to discourage over-insurance. Indeed, the rationale behind the
incorporation of “other insurance” clause in fire policies is to prevent over-insurance and thus avert the perpetration of
fraud. When a property owner obtains insurance policies from two or more insurers in a total amount that exceeds the
property’s value, the insured may have an inducement to destroy the property for the purpose of collecting the
insurance. The public as well as the insurer is interested in preventing a situation in which a fire would be profitable to
the insured.

GULF RESORT,INC VS PHILIPPINE CHARTER INSURANCE CORPORATION


G.R. No. 156167 May 16, 2005
FACTS :
Gulf Resorts is the owner of the Plaza Resort situated at Agoo, La Union
and had its properties in said resort insured originally with the American
Home Assurance Company (AHAC). In the first 4 policies issued, the
risks of loss from earthquake shock was extended only to petitioner’s
two swimming pools. Gulf Resorts agreed to insure with Phil Charter the
properties covered by the AHAC policy provided that the policy wording
and rates in said policy be copied in the policy to be issued by Phil
Charter. Phil Charter issued Policy No. 31944 to Gulf Resorts covering
the period of March 14, 1990 to March 14, 1991 for P10,700,600.00 for a
total premium of P45,159.92. the break-down of premiums shows that
Gulf Resorts paid only P393.00 as premium against earthquake shock
(ES). In Policy No. 31944 issued by defendant, the shock endorsement
provided that “In consideration of the payment by the insured to the
company of the sum included additional premium the Company agrees,
notwithstanding what is stated in the printed conditions of this policy due
60
to the contrary, that this insurance covers loss or damage to shock to
any of the property insured by this Policy occasioned by or through or in
consequence of earthquake. In Exhibit "7-C" the word "included" above
the underlined portion was deleted. On July 16, 1990 an earthquake
struck Central Luzon and Northern Luzon and plaintiff’s properties
covered by Policy No. 31944 issued by defendant, including the
two swimming pools in its Agoo Playa Resort were damaged.
Petitioner advised respondent that it would be making a claim under its
Insurance Policy 31944 for damages on its properties. Respondent
denied petitioner’s claim on the ground that its insurance policy only
afforded earthquake shock coverage to the twoswimming pools of the
resort. The trial court ruled in favor of respondent. In its ruling,
the schedule clearly shows that petitioner paid only a premium of
P393.00 against the peril of earthquake shock, the same premium it had
paid against earthquake shock only on the two swimming pools in all the
policies issued by AHAC.
ISSUE: Whether or not the policy covers only the two swimming
pools owned by Gulf Resorts and does not extend to all properties
damaged therein
HELD: Yes, All the provisions and riders taken and interpreted together,
indubitably show the intention of the parties to extend earthquake shock
coverage to the two swimming pools only. Aninsurance premium is the
consideration paid an insurer for undertaking to indemnify the insured
against a specified peril. In fire, casualty and marine insurance, the
premium becomes a debt as soon as the risk attaches. In the subject
policy, no premium payments were made with regard to earthquake
shock coverage except on the two swimming pools. There is no mention
of any premium payable for the other resort properties with regard to
earthquake shock. This is consistent with the history of
petitioner’sinsurance policies with AHAC.

Landicho vs. GSIS [G.R. No. L-28866 March 17, 1972]

FACTS:

On June 1, 1964, the GSIS issued in favor of Flaviano Landicho, a civil engineer of the Bureau of Public Works, stationed
at Mamburao, Mindoro Occidental, optional additional life insurance policy No. OG-136107 in the sum of P7,900. xxx

Before the issuance of said policy, Landicho had filed an application, by filing and signing a printed form of the GSIS on
the basis of which the policy was issued. Paragraph 7 of said application States: 7. xxx I hereby agree as follows: xxx c.
That this application serves as a letter of authority to the Collecting Officer of our Office thru the GSIS to deduct from my
salary the monthly premium in the amount of P33.36, beginning the month of May, 1964, and every month thereafter
until notice of its discontinuance shall have beenreceived from the System; . d. That the failure to deduct from my salary
the month premiums shall not make the policy lapse, however, the premium account shall be considered as
indebtedness which, I bind myself to pay the System; . e. That my policy shall be made effective on the first day of the
month next following the month the first premium is paid; provided, that it is not more ninety (90) days before or after
the date of the medical examination, was conducted if required."

While still an employee of the Bureau of Public Works, Mr. Landicho died in an airplane crash on June 29, 1966. Mrs.
Landicho, in her own behalf and that of her co-plaintiffs and minor children, Rafael J. and Maria Lourdes Eugenia, filed
with the GSIS a claim for P15,800, as the double indemnity due under policy No. OG-136107. GSIS denied the claim,
61
upon the ground that the policy had never been in force because, pursuant to subdivision (e) of the above-quoted
paragraph 7 of the application, the policy "shall be ... effective on the first day of the month next following the month the
first premium is paid," and no premium had ever been paid on said policy. The Lower Court decided in favor of the
petitioner. GSIS appealed to the Supreme Court.

ISSUE:

WON the insurance policy in question has ever been in force, not a single premium having been paid thereon.

RULING: Lower Court decision is sustained.

(T)he language, of subdivisions (c), (d) and (e) is such as to create an ambiguity that should be resolved against the party
responsible therefor — defendant GSIS, as the party who prepared and furnished the application form — and in favor of
the party misled thereby, the insured employee. Indeed, our Civil Code provides: The interpretation of obscure words or
stipulations in a contract shall not favor the party who caused the obscurity. 2

This is particularly true as regards insurance policies, in respect of which it is settled that the " "terms in an insurance
policy, which are ambiguous, equivocal, or uncertain ... are to be construed strictly and most strongly against the insurer,
and liberally in favor of the insured so as to effect the dominant purpose of indemnity or payment to the insured,
especially where a forfeiture is involved" (29 Am. Jur., 181), and the reason for this rule is the "insured usually has no
voice in the selection or arrangement of the words employed and that the language of the contract is selected with great
care and deliberation by experts and legal advisers employed by, and acting exclusively in the interest of, the insurance
company." (44 C.J.S., p. 1174.) 3.

The equitable and ethical considerations justifying the foregoing view are bolstered up by two (2) factors, namely: (a) The
aforementioned subdivision (c) states "that this application serves as a letter of authority to the Collecting Officer of our
Office" — the Bureau of Public Works — "thru the GSIS to deduct from my salary the monthly premium in the amount of
P33.36." No such deduction was made — and, consequently, not even the first premium "paid" — because the collecting
officer of the Bureau of Public Works was not advised by the GSIS to make it (the deduction) pursuant to said authority.
Surely, this omission of the GSIS should not inure to its benefit. .

(b) The GSIS had impliedly induced the insured to believe that Policy No. OG-136107 was in force, he having been paid by
the GSIS the dividends corresponding to said policy. Had the insured had the slightest inkling that the latter was not, as
yet, effective for non-payment of the first premium, he would have, in all probability, caused the same to be forthwith
satisfied.

WHEREFORE, the decision appealed from should be, it is hereby affirmed, with costs against the defendantappellant,
Government Service Insurance System. It is so ordered. .

Malayan Insurance Corp vs CA G.R. 119599 March 20, 1997

J. Romero

Facts:

TKC Marketing imported 3,000 metric tons of soya from Brazil to Manila. It was insured by Malayan at the value of
almost 20 million pesos. The vessel, however, was stranded on South Africa because of a lawsuit regarding the
possession of the soya. TKC consulted Malayan on recovery of the amount, but the latter claimed that it wasn’t covered

62
by the policy. The soya was sold in Africa for Php 10 million, but TKC wanted Malayan to shoulder the remaining value of
10 million as well.

Petitioner filed suit due to Malayan’s reticence to pay. Malayan claimed that arrest by civil authorities wasn’t covered by
the policy. The trial court ruled in TKC’s favor with damages to boot. The appellate court affirmed the decision under the
reason that clause 12 of the policy regarding an excepted risk due to arrest by civil authorities was deleted by Section 1.1
of the Institute War Clauses which covered ordinary arrests by civil authorities. Failure of the cargo to arrive was also
covered by the Theft, Pilferage, and Non-delivery Clause of the contract. Hence this petition.

Issues:

1. WON the arrest of the vessel was a risk covered under the subject insurance policies.

2. WON the insurance policies must strictly construed against the insurer.

Held: Yes. Yes. Petition dismissed.

Ratio:

1. Section 12 or the "Free from Capture & Seizure Clause" states: "Warranted free of capture, seizure, arrest, restraint or
detainment, and the consequences thereof or of any attempt thereat… Should Clause 12 be deleted, the relevant current
institute war clauses shall be deemed to form part of this insurance.”

This was really replaced by the subsection 1.1 of section 1 of Institute War Clauses (Cargo) which included “the risks
excluded from the standard form of English Marine Policy by the clause warranted free of capture, seizure, arrest,
restraint or detainment, and the consequences thereof of hostilities or warlike operations, whether there be a
declaration of war or not.”

The petitioner’s claim that the Institute War Clauses can be operative in case of hostilities or warlike operations on
account of its heading "Institute War Clauses" is not tenable. It reiterated the CA’s stand that “its interpretation in recent
years to include seizure or detention by civil authorities seems consistent with the general purposes of the clause.” This
interpretation was regardless of the fact whether the arrest was in war or by civil authorities.

The petitioner was said to have confused the Institute War clauses and the F.C.S. in English law.

“It stated that "the F.C. & S. Clause was "originally incorporated in insurance policies to eliminate the risks of warlike
operations". It also averred that the F.C. & S. Clause applies even if there be no war or warlike operations. In the same
vein, it contended that subsection 1.1 of Section 1 of the Institute War Clauses (Cargo) "pertained exclusively to warlike
operations" and yet it also stated that "the deletion of the F.C. & S. Clause and the consequent incorporation of
subsection 1.1 of Section 1 of the Institute War Clauses (Cargo) was to include "arrest, etc. even if it were not a result of
hostilities or warlike operations."

The court found that the insurance agency tried to interpret executive and political acts as those not including ordinary
arrests in the exceptions of the FCS clause , and claims that the War Clauses now included executive and political acts
without including ordinary arrests in the new stipulation.

“A strained interpretation which is unnatural and forced, as to lead to an absurd conclusion or to render the policy
nonsensical, should, by all means, be avoided.”

63
2. Indemnity and liability insurance policies are construed in accordance with the general rule of resolving any ambiguity
therein in favor of the insured, where the contract or policy is prepared by the insurer. A contract of insurance, being a
contract of adhesion, means that any ambiguity should be resolved against the insurer.

Western Guaranty v CA G.R. No. 91666 July 20, 1990

J. Feliciano

Facts:

Priscilla Rodriguez was struck by a bus owned by De Dios. She was hospitalized and her face was permanently disfigured.
Western Guaranty, the insurance company of the bus line, was obliged to pay due to the bodily injury caused by the bus.
Rodriguez was able to earn a money judgment from the court to the tune of 3000 for actual damages, 1500 for loss of
earning capacity, and 20000 for moral damages and attorney’s fees. De Dios filed a complaint against Western to
indemnify the amount. Western lost the case in the appellate court, hence this petition.

Issue: Is Western liable for paying loss of earnings, moral damages and attorney's fees even if these items are not among
those included in the Schedule of Indemnities set forth in the insurance policy.

Held: Yes. Petition dismissed.

Ratio:

The policy states:

Section 1. Liability to the Public — Company will, subject to the Limits of Liability, pay all sums necessary to discharge
liability of the insured in respect of —

(a) death of or bodily injury to or damage to property of any passenger as defined herein.

There was also a schedule of indemnities that specified a certain amount for a certain type of injury as well as hospital
service payments.

In this case, the limits on the amount payable for certain kinds of expenses were not considered by the court as
“excluding liability for any other type of expense or damage or loss even though actually sustained or incurred by the
third party victim.”

The court noted that the limits of the liability was at 50,000 per person per accident. Construing this with section 1
means that all kinds of damages allowable by law were also to be covered by the policy once it was shown that liability
has arisen.

The schedule of indemnities was not a closed enumeration of the kinds of damages Western can award.

Western should have used far more specific language, not the “pay all sums necessary to discharge liability” clause.

64
Insurance contracts must be read by the courts with a jaundiced eye to prevent the insurer from escaping from its
obligation. Also, contracts of adhesion such as policies msut be construed against the party who made them, in this case
western.

TAURUS TAXI CO., INC., FELICITAS V. MONJE, ET AL., plaintiffs-appellees,vs.THE CAPITAL INSURANCE & SURETY CO., INC.,
defendant-appellant.

FACTS:(1) Alfredo Monje was employed as a taxi driver of the Taurus Taxi.

(2) On December 6, 1962, the taxi he was driving collided with a Transport Taxicab atthe intersection of Old Sta. Mesa
and V. Mapa Streets, Manila, resulting in his death.

(3) Commercial Vehicle Comprehensive Policy was subsisting at the time of theaccident.The Policy states that the amount
for which each passenger, including thedriver, is insured is P5,000.00.

(4) The Indorsement No. 1 which forms part of the policy was given to Petitoner Felicitasand his children.

(5) Taurus Taxi made representation " for the payment of the insurance benefit whichcorresponds to the wife and
children". Defendant refused to pay.

(6) DEFENDANTS CONTENTION:that in view of the fact that the deceased Alfredo Monje was entitled to indemnity
underANOTHER INSURANCE POLICY issued by Ed. A. Keller Co., Ltd., the heirs of the saiddeceased are not entitled to
indemnity under the insurance policy issued by appellant forthe reason that the latter POLICY contains a stipulation that
"the company will indemnifyany authorized driver provided that such authorized driver is not entitled to indemnityunder
any other policy.

(2)RTC: In favor of the plaintiffs. Order the defendant pay the plaintiff. Defendant wouldseek to escape liability on the
plea that the workman's compensatio. CA: Affirmed.

ISSUE:

(1)Whether or not the defendant-appellant will indemnify any authorized driver providedthat [he] is not entitled to any
indemnity under any other policy, it being shown that thedeceased was paid his workman's compensation from another
insurance policy, shoulddefeat such a right to recover under the insurance contract subject of this suit?

(2) Whether or not the plaintiffs committed breach of policy condition when they were joined by Taurus?(Basis is one of
the conditions in the policy which states that "no admission, offer,promise or payment shall be made by or on behalf of
the insured without the writtenconsent of the Company x x x)

HELD:(1)Yes. The petitioner may still recover or still entitled to indemnification from thedefendant.

(2)No. There is no breach of policy.

RATIO:(1)If there is doubt concerning the liability of the defendant insurance firm, it should beresolved against the
insurance and in favor of the insured because a contract ofinsurance couched in language chosen by the insurer is, if
open to the constructioncontended for by the insured, to be construed most strongly, or strictly, against theinsurer and
liberally in favor of the contention of the insured, which means in accordancewith the rule contra proferentem.

65
(2)The institution of the action cannot possibly be construed as an admission, offer,promise, or payment by the company,
for it merely seeks to enforce, by court action, theonly legal remedy available to it, its rights under the contract of
insurance to which it is aparty

FINMAN GENERAL ASSURANCE CORPORATION vs. CA and SURPOSA G.R. No.


100970 September 2, 1992

FACTS:

On October 22, 1986, deceased, Carlie Surposa was insured with under Finman General Teachers Protection Plan (an
accident insurance) and Individual Policy with his parents, spouses Julia and Carlos Surposa, and brothers Christopher,
Charles, Chester and Clifton, all surnamed, Surposa, as beneficiaries.

The insured died on October 18, 1988 as a result of a stab wound inflicted by one of the 3 unidentified men as he and his
cousin, Winston Surposa, were waiting for a ride on their way home along Rizal-Locsin Streets, Bacolod City.

Private respondent and the other beneficiaries of said insurance policy filed a written notice of claim with the insurance
company which denied said claim contending that murder and assault are not within the scope of the coverage of the
insurance policy.

The Insurance Commission rendered a decision holding the insurer is liable to pay the proceeds of the policy which was
affirmed by the CA.

ISSUE:

Whether the death of the insured was committed with deliberate intent which, by the very nature of a personal accident
insurance policy, cannot be indemnified

RULING:

No, the death of the insured was no commited with deliberate intent.

The generally accepted rule is that, death or injury does not result from accident or accidental means within the terms of
an accident-policy if it is the natural result of the insured's voluntary act, unaccompanied by anything unforeseen except
the death or injury. The happening, on the part of the insured is a pure accident. The insured died from an event that
took place without his foresight or expectation, an event that proceeded from an unusual effect of a known cause and,

66
therefore, not expected. Neither can it be said that where was a capricious desire on the part of the accused to expose
his life to danger considering that he was just going home after attending a festival.

Furthermore, the personal accident insurance policy involved herein specifically enumerated only ten (10) circumstances
wherein no liability attaches to insurance company for any injury, disability or loss suffered by the insured as a result of
any of the stipulated causes. The principle of " expresso unius exclusio alterius" is therefore applicable in the instant case
since murder and assault, not having been expressly included in the enumeration of the circumstances that would
negate liability in said insurance policy, cannot be considered by implication to discharge the petitioner insurance
company from liability for, any injury, disability or loss suffered by the insured.

The interpretation of obscure words or stipulations in a contract shall not favor the party who caused the obscurity (NCC
1377). Moreover,it is well settled that contracts of insurance are to be construed liberally in favor of the insured and
strictly against the insurer. Thus ambiguity in the words of an insurance contract should be interpreted in favor of its
beneficiary.

Jarque v Smith G.R. No. L-32986 November 11, 1930 J. Ostrand

Facts: The plaintiff was the owner of the motorboat Pandan and held a marine insurance policy for the sum of P45,000
on the boat, the policy being issued by the National Union Fire Insurance

Company and according to the provisions of a "rider" attached to the policy, the insurance was against the "absolute
total loss of the vessel only."

The ship ran into very heavy sea off the Islands of Ticlin, and it became necessary to jettison a portion of the cargo. As a
result of the jettison, the National Union Fire Insurance Company was assessed in the sum of P2,610.86 as its
contribution to the general average. The insurance company, insisting that its obligation did not extend beyond the
insurance of the "absolute total loss of the vessel only, and to pay proportionate salvage of the declared value," refused
to contribute to the settlement of the general average.

The present action was thereupon instituted,and after trial the court below rendered judgment in favor of the plaintiff
and ordered the company to pay the plaintiff the sum of P2,610.86 as its part of the indemnity The insurance company
appealed to this court.

Issues: 1. WON the lower court erred in disregarding the typewritten clause endorsed upon the policy, Exhibit A,
expressly limiting insurer's liability thereunder of the total loss of the wooden vessel Pandanand to proportionate salvage
charges

67
2. WON the lower court erred in concluding that defendant and appellant, National Union Fire Insurance Company is
liable to contribute to the general average resulting from the jettison of a part of said vessel's cargo.

Held: No to both. Petition dismissed.

Ratio: 1. One of the clauses of the document originally read as follows: Touching the Adventures and Perils which the
said National Union Fire Insurance Company is content to bear, and to take upon them in this Voyage; they are of the
Seas, Men-of-War, Fire, Pirates, Rovers, Thieves, Jettison, Letters of Mart and Countermart, Surprisals, and Takings at Sea.
Arrest, Restraint and Detainments, of all Kings Princes and People of what Nation, Condition or Quality so ever; Barratry
of the Master and Marines, and of all other Perils, Losses and Misfortunes, that have or shall come to the Hurt,
Detriment, or Damage of the said Vessel or any part thereof; and in case of any Loss or Misfortunes, it shall be lawful for
the Assured, his or their Factors, Servants, or assigns, to sue, labor and travel for, in and about the Defense. Safeguard,
and recovery of the said Vessel or any Charges whereof the said Company, will contribute, according to the rate and
quantity of the sum herein assured shall be of as much force and Virtue as the surest Writing or Policy of Insurance made
in LONDON. Attached to the policy over and above the said clause is a "rider" containing typewritten provisions, among
which appears in capitalized type the following clause:

AGAINST THE ABSOLUTE TOTAL LOSS OF THE VESSEL ONLY, AND TO PAY PROPORTIONATE SALVAGE CHARGES OF THE
DECLARED VALUE.

At the bottom of the same rider following the type written provisions therein set forth are the following words:
"Attaching

to and forming part of the National Union Fire Insurance Co., Hull Policy No. 1055."

It is a well settled rule that in case repugnance exists between written and printed portions of a policy, the written
portion prevails, and there can be no question that as far as any inconsistency exists, the above-mentioned typed "rider"
prevails over the printed clause it covers. Section 291 of the Code of Civil Procedure provides that "when an instrument
consists partly of written words and partly of a printed form and the two are inconsistent, the former controls the latter.

2. In the absence of positive legislation to the contrary, the liability of the defendant insurance company on its policy
would, perhaps, be limited to "absolute loss of the vessel only, and to pay proportionate salvage of the declared value."
But the policy was executed in this jurisdiction and "warranted to trade within the waters of the Philippine Archipelago
only." Here the liability for contribution in general average is not based on the express terms of the policy, but rest upon
the theory that from the relation of the parties and for their benefit, a quasi contract is implied by law. Article 859 of the
Code of Commerce is still in force and reads as follows:

68
ART. 859. The underwriters of the vessel, of the freight, and of the cargo shall be obliged to pay for the indemnity of the
gross average in so far as is required of each one of these objects respectively. The article is mandatory in its terms, and
the insurers, whether for the vessel or for the freight or for the cargo, is bound to contribute to the indemnity of the
general average. It simply places the insurer on the same footing as other persons who have an interest in the vessel, or
the cargo

therein at the time of the occurrence of the general average and who are compelled to contribute.

In the present case it is not disputed that the ship was in grave peril and that the jettison of part of the cargo was
necessary. If the cargo was in peril to the extent of call for general average, the ship must also have been in great danger,
possibly sufficient to cause its absolute loss. The jettison was therefore as much to the benefit of the underwriter as to
the owner of the cargo. If no jettison had take place and if the ship by reason thereof had foundered, the underwriter's
loss would have been many times as large as the contribution now demanded.

LIM V. SUN LIFE 41 PHIL 263

Facts: > On July 6, 1917, Luis Lim Y Garcia of Zamboanga applied for a policy of life insurance with Sunlife in the amount
of 5T. > He designated his wife Pilar Lim as the beneficiary. The first premium of P433 was paid by Lim and company
issued a ―provisional policy‖ > Such policy contained the following provisions “xx the abovementioned life is to be
assured in accordance with the terms and conditions contained or inserted by the Company in the policy which may be
granted by it in this particular case for 4 months only from the date of the application, PROVIDED that the company shall
confirm this agreement by issuing a policy on said application xxx. Should the company NOT issue such a policy, then this
agreement shall be null and void ab initio and the Company shall be held not to have been on the risk at all, but in such
case, the amount herein shall be returned. > Lim died on Aug. 23, 1917 after the issuance of the provisional policy but
before the approval of the application by the home office of the insurance company. > The instant action is brought by
the beneficiary to recover from Sun Life the sum of 5T.

11

Issue: Whether or not the beneficiary can collect the 5T.

Held: NO. The contract of insurance was not consummated by the parties. The above quoted agreement clearly stated
that the agreement should NOT go into effect until the home office of the Company shall confirm it by issuing a policy. It
was nothing but an acknowledgment by the Company that it has received a sum of money agreed upon as the first year’s
premium upon a policy to be issued upon the application if it is accepted by the Company.

When an agreement is made between the applicant and the agent whether by signing an application containing such
condition or otherwise, that no liability shall attach until the principal approves the risk and a receipt is given by the
agent, such acceptance is merely conditional and is subordinated to the company’s act in approving or rejecting; so in life
insurance a ―binding slip or receipt‖ does not insure itself.

69
Development Bank of the Philippines v CA 231 SCRA 370 March 21, 1994 GR. 109937

Facts: Juan B. Dans, together with his family applied for a loan of P500,000 with DBP. As principal mortgagor, Dans, then
76 years of age was advised by DBP to obtain a mortgage redemption insurance (MRI) with DBP MRI pool. A loan in the
reduced amount was approved and released by DBP. From the proceeds of the loan, DBP deducted the payment for the
MRI premium. The MRI premium of Dans, less the DBP service fee of 10%, was credited by DBP to the savings account of
DBP MRI-Pool. Accordingly, the DBP MRI Pool was advised of the credit. Dans died of cardiac arrest. DBP MRI Pool
notified DBP that Dans was not eligible for MRI coverage, being over the acceptance age limit of 60 years at the time of
application. DBP apprised Candida Dans of the disapproval of her late husband’s MRI application. DBP offered to refund
the premium which the deceased had paid, but Candida Dans refused to accept the same demanding payment of the
face value of the MRI or an amount

equivalent of the loan. She, likewise, refused to accept an ex gratia settlement which DBP later offered. Hence the case at
bar.

Issue: Whether or not the DBP MRI Pool should be held liable on the ground that the contract was already perfected?

Held: No, it is not liable. The power to approve MRI application is lodged with the DBP MRI Pool. The pool, however, did
not approve the application. There is also no showing that it accepted the sum which DBP credited to its account with
full knowledge that it was payment for the premium. There was as a result no perfected contract of insurance’ hence the
DBP MRI Pool cannot be held liable on a contract that does not exist In dealing with Dans, DBP was wearing 2 legal hats:
the first as a lender and the second as an insurance agent. As an insurance agent, DBP made Dans go through the motion
of applying for said insurance, thereby leading him and his family to believe that they had already fulfilled all the
requirements for the MRI and that the issuance of their policy was forthcoming. DBP had full knowledge that the
application was never going to be approved. The DBP is not authorized to accept applications for MRI when its clients are
more than 60 years of age. Knowing all the while that Dans was ineligible, DBP exceeded the scope of its authority when
it accepted the application for MRI by collecting the insurance premium and deducting its agent’s commission and
service fee. Since the third person dealing with an agent is unaware of the limits of the authority conferred by the
principal on the agent and he has been deceived by the nondisclosure thereof by the agent, then the latter is liable for
damages to him.

Enriquez vs. Sun Life Assurance Co. [GR No. 15895; November 29, 1920]

Facts: Plaintiff is estate administrator for late Joaquin Herrer. Herrer has pending application with defendant Sun Life
Assurance Co (sun Life) evidenced by a provisional receipt. The provisional receipt reads payment of Php6, 000 for life
annuity received 26 September 1917. The application was received by Sun Life head office a month after. 04 December
1917, the policy was issued in Montreal. A petition for withdrawal of application was filed by Herrer’s lawyer 18
December 1917. Herrer died 20 December. A letter from Sun Life was received 21 December stating policy was issued
and reminds the party of a notification of acceptance of the application dated 26 November. Plaintiff testified that he
had found no letter of notification from the Sun Life. Lower Court decides in favor of respondent. Appeal was taken.

Issue: Whether or not the there has been a valid offer and acceptance??

Held: None. The Civil Code provides that the acceptance made by letter binds the person making the offer only from the
date it has came to its knowledge. The contract of life annuity was not perfected. There was no satisfactory evidence that
the application acceptance came to the knowledge of Herrer.

Article 16 of the civil code provides that any deficiency in the special law shall be supplied by the Code. The Insurance
Code does not provide for law on the principle of acceptance, thus the Civil Code shall govern. Article 1262 provides that
70
consent is shown by concurrence of offer and acceptance with the thing and the consideration to the contract. The
acceptance by letter shall not bind the person making the offer except from the time It came to his knowledge.
American Courts held that acceptance of offer not actually communicated does not complete the contract but the
mailing of the acceptance. Locus Poenitrntiae is ended when acceptance has passed beyond party’s control.
Furthermore, the provisional receipt provides for conditions before a contract is deemed final. 1. Medical examination. 2.
Approval by head office of the application. 3. the company communicates approval to the applicant. In the case, there
was no letter of notification. No evidence of knowledge. Judgment reversed. Php6000 with interest is to be returned.

ANG GIOK CHIP, vs. SPRINGFIELD FIRE & MARINE INSURANCE COMPANY

FACTS:

Ang Giok Chip doing business under the name and style of Hua Bee Kong Si was formerly the owner of a warehouse
situated at No. 643 Calle Reina Regente, City of Manila. The contents of the warehouse were insured with the three
insurance companies for the total sum of P60,000. One insurance policy, in the amount of P10,000, was taken out with
the Springfield Fire & Marine Insurance Company. The warehouse was destroyed by fire on January 11, 1928, while the
policy issued by the latter company was in force. The insurance company interposed its defense on a rider in the policy in
the form of Warranty F, fixing the amount of hazardous good that can be stored in a building to be covered by the
insurance. They

claimed that Ang violated the 3 percent limit by placing hazardous goods to as high as 39 percent of all the goods stored
in the building. His suit to recover was granted by the trial court.

ISSUE:

Whether a warranty referred to in the policy as forming part of the contract of insurance and in the form of a rider to the
insurance policy, is null and void because not complying with the Philippine Insurance Act

RULING OF THE COURT The Insurance Act, Section 65, taken from California law, states:

"Every express warranty, made at or before the execution of a policy, must be contained in the policy itself, or in another
instrument signed by the insured and referred to in the policy, as making a part of it."

Warranty F, indemnifying for a value of Php 20,000 and pasted on the left margin of the policy stated:

It is hereby declared and agreed that during the currency of this policy no hazardous goods be stored in the Building to
which this insurance applies or in any building communicating therewith, provided, always, however, that the Insured be
permitted to stored a small quantity of the hazardous goods specified below, but not exceeding in all 3 per cent of the
total value of the whole of the goods or merchandise contained in said warehouse, viz; . . . .

Also, the court stated a book that said, "any express warranty or condition is always a part of the policy, but, like any
other part of an express contract, may be written in the margin, or contained in proposals or documents expressly
referred to in the policy, and so made a part of it."

“It is well settled that a rider attached to a policy is a part of the contract, to the same extent and with like effect as it
actually embodied therein. In the second place, it is equally well settled that an express warranty must appear upon the
face of the policy, or be clearly incorporated therein and made a part thereof by explicit reference, or by words clearly
evidencing such intention.”

The court concluded that Warranty F is contained in the policy itself, because by the contract of insurance agreed to by
the parties it was made to be a part. It wasn’t a separate instrument agreed to by the parties.

71
The receipt of the policy by the insured without objection binds him. It was his duty to read the policy and know its
terms. He also never chose to accept a different policy by considering the earlier one as a mistake.

72

Potrebbero piacerti anche